Anda di halaman 1dari 69

NAME:----------------------------------------------

DATE:-------------------------------
This bank of questions was made by a group of doctors who are looking to pass the Evaluating Exam
& in the mean time looking to help all the others to achieve there aims ... these questions are a
collection from the previous Evaluating Exams that came in the past few years ... so take it serious &
be sure that most of the questions will be in your Exam Inshallah ... so pray to the !od may help us
passing this Exam "
1. Which of the following is correct regarding Wernikes Encephalopathy :
-Often develops into Korsakof's even when treated
-It has a mortality rate of ! " to #! " if $ntreated
-%he treatment is thiamine 1!! mg. &O daily for ' days
-%he symptoms incl$de (istagm$s ata)ia and opisthotonos
-(one of the a*ove +
,. Which is the most serio$s complication of the s$pra condilar fract$re of the -$mer$s .
-/ compartment syndrome of forearm
-0ail$re to heal
--ealing in a non anatomic position
-In1$ry into the median nerve.
-&ermanent restriction of the El*ow motion.
2. Which one of the following descriptors of a diagnostic test is Infl$enced *y the prevalence of the
disease *eing tested for :
-3pecificity
-3ensitivity
-/cc$racy
-&ositive predictive val$e
-4elia*ility
5. / 52 years 6 admitted for Emergency 7astrectomey present conf$sion on the 2rd post-Op day.
complains of lack of sleep d$e to cockroaches on the ceiling he is noted to *e flashed 8 tremel$s *y
the n$rses d$ring the day .. the most likely pro*lem is :
-&ost-Op Electrolites In*alance
-&aranoid 3chi9ophrenia
-:epression &sychosis
-:eleri$m %remons
-/no)ic *rain system
'. &t. with known type II :6. treated with ;lorpropamide present to E: in comma .. *lood s$gar <
1 mmol = >. .. yo$ give the &t 1 amp$le of :'!W 8 the &t wakes $p promptley . what is the ne)t
appropriate management :
-7ive another amp$le then discharge.
-7ive , amp$les then discharge.
-7ive another amp$le then o*serve the pt for ? h. in the E:.
-7ive one more amp$le @ admit the &t. 8 start IA. 7l$cose Inf$sion with freB$ent ongoing gl$cose
meas$rement
?. &regnant 2? weeks with vaginal watery discharge @ management :
-do sterile spec$l$m e)am
. Car*it$rate withdrawal < conv$lsion
#. 4etrovarted $ter$s complain < asymptomatic
D. 7' &5 post-delivery@ postpart$m hemorrhage@ most likely ca$se is :
-Eterine atony
1!. /nal skin tag in child associated with @
-chronic /nal 0iss$re
11. ;hild with a*dominal pain attacks @ drowning his legs $p @ m$co$id *loody stool. :iagnosis .
-Int$ss$sception
1,. 1st sign of foot gangrene is < rest pain
12. &t. 6 pain in *oth lower lim*s with week popletial artery p$lsation @ management .
transverte*ral angiogram
15. *asket *all player averted his ankle 1oint d$ring 1$mping at match @ on E)amination &ain with
Increased Aario$s range. 6anagement .
-4epair ligament s$rgery
-Celow knee cast
1'. clean wo$nd c$t with laceration 8 incomplete section of nerve @ management .
-3$t$re of wo$nd Immediately
->eave the wo$nd open
1?. pain on walking increase in ,nd 8 2rd metatarsal *one of forefoot. :iagnosis .
-3tress fract$re
-3esamoid *one
1. &t. ?# years going to elective s$rgery @ he has 5 mo. history of chest pain 8 got same attack at
night *efore the day of s$rgery @ what do $ do .
->ignocaine *efore s$rgery
-%rinatreate d$ring s$rgery
-3end him for I;E
-&ostpone the operation 8 check f$nction of the coronaries
1#. collage st$dent 0 ,! years presented with lower a*dominal pain @ &E *ilateral lower a*dominal
tenderness @ fe*rile Aaginal e)am : tender with cervical mo*ili9ation @ pregnant test F-G. :iagnosis .
-/c$te salpengitis
-Ectopic pregnancy
-E%I
1D. vaginal *leeding @ stooped at the day of admission @ pregnancy test is FHG E3 shows empty $ter$s
8 left adne)al mass. :iagnosis .
-Ectopic pregnancy
,!. yo$ng &t with history of non prod$ctive co$gh *$t clinically well .. ;-I ray shows *ilateral *asal
Infiltration .. :iagnosis. .
-6ycoplasma &ne$monia
,1. &t with Elcer in sole of the foot @ at ,nd 8 2rd metatarsal 1oint I 4ay necrotic 8 destroyed part
of ,nd 8 2rd metatarsal *one .. what to do .
-4adical de*ridment
-/*ove knee amp$tation
-:aily sterile dressing
-Oral anti*iotic
,,. &ro*lem which ca$se most work days < *ack pain
,2. (icotinic skin patchy is contraindicated in:
-&regnancy
-;A/
-Ischemic -eart disease
-/lcoholic
(C. Fnot s$reG
,5. &sychotherapy is s$perior to medication in:
-3chi9ophrenia
-Cipolar disorder
-/lcoholic withdrawal
-:ysthimia
,'. mother worried a*o$t her child *eca$se of history of myopathy in family .. what Investigation to
*e done 1st .
-;&K
-6$scle *iopsy
-(erve *iopsy
-E67
,?. &t on >ithi$m therapy *ecame weak@ lethargic@ Intolerant to heat. what Investigation m$st *e
done .
->ithi$m level
-C&
-%hyroid 0$nction test
,. s$rgical n$rse ,' years old is concerned that she is loosing her mind@ for the past ? months she is
*een preocc$pied with contamination@ on his ward she has *een not to$ching any patient @door kno*s
...etc.
3he was also washing her hand e)cessively@ which of the following treatments will help in red$cing
her preocc$pation and hand washing .
-0l$)etine
->ora9epam
-&erphena9ine
-Insight-oriented psychotherapy
-(ifedipine
,#. 6 '! years with &rostatic ;ancer with *one metastasis. %reatment .
radical prostatectomy
-4adiotherapy
--ormonal therapy in the form of total androgenic *lockage
-IA chemotherapy
,D. &t with family history of $rticaria@ presented with $rticaria ... investigation revealed ;1 esterase
inhi*itor deficiency ... :iagnosis. .
--ereditary /ngioedema
2!. thickened $pper >t. lip with mild vesicles ... :iagnosis .
-Erticaria
-/ngioedema
--3A infection
21. Erithemato$s A$lva with whitish lesion of ;andida of 7roin 8 3atellite lesions .. what is the
predisposing disease .
-:6
-;/. v$lva
->ichen sclerosis
2,. &t with s$dden co$gh 8 pain in the 4t. side of the chest .. yo$ s$spect aspiration of foreign
*ody .. what finding of the ;-I ray .
--yperl$cent 4t. side
22. :efrentiaion *etween 6a)ilary 8 Jigomatic fract$res .......
25. calc$lation of the effectiveness of a vaccine
Incidence (on Aaccinate - Incidence Aaccinated
--------------------------------------------------------------- ) 1!!
Incidence (on Aaccinate
2'. child chalking d$ring eating .. he *ecame ;yanotic@ agitated@ with forcef$l *reathing .. 1st action
to do is :
--it him from his *ack while head downward
-Introd$ce yo$r finger in his mo$th
2?. 1 years 0. at stage 2 %anner still not menses .. management .
-E)amination of the pelvis
-;hromosome analysis
-Estrogen &rogesterone level
-4eass$rance
2. asthmatic &t. presented in the E4 with :yspnea @ $sed to inhaled sal*$tamol with no
Improvement.. &E shows dyspnea slit rhonchi at the $pper chest 8 neck ... management. .
-IA. fl$id H /minophine
-;oricosteroids Inhaled
-Inhaled 3al*$tamol H IA hydrocortisone H IA 0l$id
2#. /lfa fetoprotein Increased in
-6enengomyelocele
-4enal /genesis
-:own 3yndrome
2D. pt with a*normal pap smear .. what is the ne)t step .
-;olposcopy
5!. ?! years 0. presented with ' ) ' cm. adne)al mass .. management. .
-3$rgery
-;hemotherapy
-4adiotherapy
51. &t. fo$nd $nconscio$s on the floor *ehind the *ar@ alcoholic@ E4 agitated semiconscio$s ... &E
shows laceration on the head@ all the lim*s can *e moved@ all the others are normalK what is yo$r
action.
-;% of the head
-IA. fl$id H O*servation
-:ischarge
5,. &t. fail down from the ,nd floor on his *ack .. the following will *e part of the Initial
management EI;E&% :
-;ervical collar
-IA. fl$id
-3pine I 4ay
-Erinary catheter
52. tertiary prevention is
-4eha*ilitation.
55. in the E4. yo$ng &t treated with &rocainamide his C&. decreased to #!L?! yo$ tried another time
with &rocainamide .. his C&. still decreasing .. what to do .
-Cen9tropine
-3aline perf$sion
-:igitalis
-:efi*rillation
-Int$*ation
(C. the &t. had /rrhythmia with hypertension FthatMs why we gave him procainamideG 8 *eca$se its
not decreasing we have to defi*rillate him ...
5'. 0 ,2 years present with fi*roadenoma of the >t. *reast in the lower medial B$adrant of the *reast
all are possi*le EI;E&% :
-7ive Estrogen
-7ive &rogesterone
-0ine needle aspiration will *ring clear liB$id
-6ammography can show a*normality
-%he growth of the %$mor following cycles
5?. 0. fe*rile with t$mor in $pper 4t. B$adrant of the an$s .. the mass is painf$l 8 red .. what to do .
hot *andages
-;old sit9 *aths
-3$rgery
-3ystemic /nti*iotics
-%opical /nti*iotics
5. child with 1$venile poliposis .. possi*le complication :
-:iarrhea
-&ainless *leeding
-&ainf$l *leeding
-6alignancy FcancerG
-(on of the a*ove
5#. a mother *ro$ght hear 1, years da$ghter who present repetitive E%I temp. 2#.' @ IA& 8
cystography are normal .. what is the *est test to do :
-Erine c$lt$re
-E3 8 voiding cystography
-Clood c$lt$re
-;ystoscopey
-(one of the a*ove
5D. fi*roadenoma of the *reast what is false :
-%he most common *enign t$mor
-0ine needle aspiration *ring clear fl$id
-%r$e solid t$mor
-%$mor easy to find *eca$se encaps$lated
(C. check it alone
'!. 0 pregnant ,# years .. with noct$rnal n$m*ness in hands 8 forearm which wakes hear $p for 2
months .. she also have diffic$lty grasping o*1ects .. what is the most likely etiology .
-:ermatomiosis
-/*r$ptio placenta
-6$ltiple sclerosis
-;arpal %$nnel 3yndrome
--yperventilation syndrome
'1. , years old with heart fail$re with high *lood press$re treated for long time. -e was *ro$ght to
E 4 for hypotension #!L?! -4 11!@ what to give .
-&rocaine
->idocaine
-;ardioversion
-:igitaline
-0$rosemide
',. &ost-op. of &ancreas .. what is the ca$se of hypovolemic shock .
-Initial necrosis of ac$te pancreatitis has prod$ced tripsine
-3eptic shock
->oss of liB$ids
-3ide effect of anesthesia
(C. not s$re
'2. ?# years 0. with history of -ysterectomy@ came to see $ *eca$se of mict$ration tro$*le at effort@
no digestive tro$*le@ she has a feeling of a painless mass intravaginaly@ she has to try twice in order
to void her *ladder what is the diagnosis .
-4ectocele
-;ystocele
-&rolapse F&rosedentiaG
-Erethral sphincter spasm
-&ost-s$rgical strict$re
'5. yo$ng 6. feels s$dden crack in his calf@ what is the *est clinical sign to diagnose /chilles tendon
r$pt$re .
-:ecreased dorsal fle)ion
-Impossi*ility to walk on the toes
-Increased passive dorsiflection of the foot
-3B$ee9ing calf does not passively planter fle) foot
''. farmer 5 years 6. present with *ilateral semetrical nea$rosensorial defenses @ &E is normal .
diagnosis .
-/$tosclerosis
-&rofessional defenses
-/c$stic ne$roma
-&res*iac$sis
-;irc$latory deficit
(C. 6ost common ca$se of hear losing in elderly is pres*iac$sis
'?. 0. 5' years with decreased vis$al ac$ity@ no pain @ no tears. what do $ e)am first .
-%onometry
-4efraction
-/ngiography
-(e$rologic test
-4he$matoid factor screening
'. C$rned Infant ,nd degree in the $pper lim*@ what do $ do .
-:e*rid 8 skin graft.
-:e*rid 8 *andage
-:e*rid 8 topic cream
-;lean the wo$nd 8 systemic anti*iotic
->ocal care only
'#. ;yclist attacked *y *ee comes to emergency with hypotension #!L?! @ -eart rate 11' .. what do $
do first .
-/ntihistamine -1 8 -,
-3aline perf$sion
-Epinephrine In1ection
-Int$*ation
-IA. steroid
(C. we also give it *y s$*c$taneo$s or I6. 8 in children IA or Endotracheal t$*e
'D. child ' years *itten *y the neigh*ors dog .. the dog received all the vaccines. what do $ do .
-O*serve the dog 1! days 8 anti ra**i ser$m
-O*serve the dog 8 anti ra**is ser$m 8 vaccine
-Kill the dog
-Aaccinate the kid
-O*serve the dog
?!. 0. ,# years present with chronic rhinorrhea @ in the e)am yo$ fo$nd m$co$sal nasal atrophy ..
diagnosis .
-3in$sitis
-;ocaine intake
-(asal poliposis
-/llergic rhinitis
?1. 0. 5' years *ack from a plane travel @ complains vertigo@ tenit$s@ moderate hearing loss@ C& is
1?!L11! .. :iagnosis .
--ypertensive crise
-6iners disease
-6igraine
-/c$stic ne$roma
-Carotar$ma
?,. what is the most common ca$se of l$ng a*scess in post-Op pt.
-Cacterial discrimination from operative site
-/telectasia
-&ne$monia
-/spiration
?2. 6. ,' years when he goes to toilet@ he complains of painf$l defecation in the anal region@ at the
e)am yo$ will find *l$ish mass at the anal margin.. what is the :iagnosis .
--emorrhoids internal sclerosed
--emorrhoids e)ternal strang$lated
--emorrhoids e)ternal throm*osed
-&yelonedle cyst
-&oliposis
?5. all the following t$mors favori9ed *y %OC/;;O E)cept...
-Cladder
-Esophag$s
->iver
-;ervi)
->$ng
?'. ' years child have *een *eaten *y a dog .. he had a deep wo$nd@ the dog is in a safe place .. what
do $ do .
-:isinfect 8 clean the wo$nd 8 p$t *andage
??. *reast feeding women with :A%. treatment. .
-Warferin
-IA. heparin
-/ntistreptocinase
-Candage
?. immediate post-op presents s$dden hall$cination @ agitation 8 pretend that was miss treated ..
:iagnosis .
-&sychosis
-:epression
-&ost-op deliri$m
-/nalgesic withdrawal syndrome
?#. a pt cons$lt $ for painless testic$lar swelling @ transil$mination was F-G after minor tra$ma ...
:iagnosis .
-3permatocele
-Epidedimitis
-Aaricocele
--ydrocele
-7erminoma
?D. 0. 5 years present with s$dden strong pain of the 4t. $pper B$adrant with fever 2#.' vomiting@
no 1a$ndice ... :iagnosis .
-;olid$tal o*str$ction
-;/ of pancreas
-/c$te cholicystitis
-/c$te hepatitis
-Ciliary lithiasis
!. &t refared to $ *y his dentist *eca$se of whitish lesion at the *ase of the tong$e .. what is yo$r
attit$de .
-Ciopsy
-Identification of germs
-/sk for a correction of his dental prostheses
-%reat him *y anti micotic oral *ath
1. yo$ng *oy present with freB$ent episode of rhinitis with p$r$lent discharge 8 fever@ from a
ma)illary sin$s@ what is the most likely complication if $ntreated .
-&$r$lent meningitis
-Ethmoidal sin$sitis
-Or*ital cell$litis
-0rontal sin$sitis
,. 0 pt. &E. painless mo*ile mass on 4t. $pper B$adrant witho$t fever or Na$ndice@ what is the most
likely diagnosis .
-%$mor of >t. inferior lo*e of the >iver
--ydrocholecyst
-%$mor of s$perior pole of the 4t. kidney
->iver metastasis
2. child 12 years o*ese present with painf$l 4t. knee 8 pain of 4t. hip with diffic$lty walking 8
red$ced a*d$ction 8 Internal rotation @ what is yo$r diagnosis .
-N$venile /rthritis
-3eptic arthritis of the hip
-/vasc$lar necrosis of femoral head
-;artilage dysplasia
-3lipped capital epiphysis
5. &t. fell on his hand @ Emergency I 4ay was normal @ *$t the pt. complain from painf$l wrist at
a*d$ction. management. .
-;ast
-E)plain him that the pain will disappear after , weeks@ its normal.
-Candage
-3hort cast pl$s I 4ay in 1' days
-(3/I:'s
(C. diagnosis is 3caphoid fract$re 8 its very diffic$lt to see it in I 4ay.
'. Indication of elastic *and in -emorrhoids is :
-%hrom*osed -emorrhoids
-%ype II *leeding Internal -emorrhoids
-E)ternal -emorrhoids
-/ngioma
?. ?' years o*ese &t. complains of strong 8 s$dden mid a*dominal pain radiating to >t. flank@ what
is yo$r diagnosis .
-/c$te pancreatitis
-;holecystitis
-Ereteral stone
-6esenteric Infarct$s
-4$pt$re of a*dominal aorta
. &t. complains of 4t. $pper B$adrant pain 8 fever @ &E shows slight a*dominal tenderness@ what
investigation do $ do to comfirme the diagnosis .
-Ciopsy
-Cari$m meal
-&eritoneal /spiration
-4ectosigmoidoscopy
-Echography
#. most common complication of Epid$ral /nesthesia in a pregnant woman:
-0etal hypo)ia
--ypotension
-:ecrease in contraction
-/rrest of la*or
D. ;hild p$lled *y his mother from his arms .. &E shows that one arm is add$cted 8 the forearm is
on pronation. what is the most likely diagnosis .
--$meral fract$re
->a)ation of the Inferior e)tremity of the $lna
-3$*l$)ation of $lnar head
-4$pt$re of rotator c$ff
#!. defense mechanism in pho*ia < displacement@ avoidance
#1. defense mechanism in *orderline &:. < splitting
#,. &t. doing the opposite of what he *elieves is good < reaction formation.
#2. &t. on lithi$m therapy complain of fatig$e @ increased weight @ heat intolerance @ what test wo$ld
yo$ order .
-%hyroid stim$lating hormone level test
#5. &t. presented with p$lmonary edema 8 C&. 1#!LD' @ what is the *est treatment .
-0$resemide
#'. 5 years 0. with 1a$ndice 8 history of na$sea 8 vomiting @ anore)ia 8 depression @ she is on
4ifampicine@ I(-@ &yrido)ine. &E -epatomegaly H tenderness@ Investigation reveal :37O%@ 37&%
increased ) '@ />& slightly increased . the most likely diagnosis is :
-:r$g ind$ced hepatitis
-Airal hepatitis
-/c$te pancreatitis
->iver ;/.
#?. &t. complain from vis$al spatial defect @ where is the lesion .
-4t. cere*ral hemisphere
->t. cere*ral hemisphere
#. 5 years /frican *oy on septrine for tonsillitis presented with Na$ndice @ Investigations : -*. D.#
gm " @ retic$locytes co$nt # " @ -*. electrophoresis : -*./ . what is the most likely diagnosis .
-3ickle cell anemia
-%halassemia
-3pherocytosis
-7?&: deficiency
##. 0. ?' years present with rec$rrent painless lower 7I *leeding@ colonoscopy@ gastroscopy@ *ari$m
enema@ endoscopy 8 $pper 7I series all are normal@ what is the dig. .
-4t. colon angiodysplasia
#D. 0. '! years operated for cholicistectomy presented with heavy wo$nd *leeding @ she gives yo$ a
history of massive *leeding when she had a tooth e)traction @ what will yo$ find .
-Increased &%% H Increased C%
-Increased &%% H decreased C%
-(ormal &%% H Increased C%
(C. diagnosis is Aon Wille*rand disease
D!. pregnant woman with a*r$ptio placenta@ developed echymosis 8 *leeding @ yo$r diagnosis is
:I;. what is the most specific test to confirm it .
-Increased 0:& Ffi*rine degradation prod$ctG
D1. 4%/ Froad traffic accidentG &t. with m$ltiple fract$re 8 tra$ma@ ,5 h later he develop
tachypnea@ &;O, < 2,@ &aO, < '!. I 4ay shows *ilateral infiltrates what is the ne)t step to do .
->$ng scan
-Aentilator
D,. all the following are primary prevention e)cept :
-Imm$ni9ation
--ealth related ed$cation
-&ap smear
D2. pregnant lady developed postpart$m hemorrhage after vac$$m delivery @ what is the most likely
ca$se .
-Eterine atony
-:I;
D5. primigravida after delivery of a 5!!! gm *a*y@ after a prolonged la*or co$ld not $rinate. What is
the most likely ca$se .
-Cladder atony
D'. ;hild 4%/ present with $rethral *leeding 8 $rinary retention. 6anagement .
-4etrograde $rethrogram
D?. 6. ,' years 4%/@ 5 h. >ater developed 4t. 0lank pain with heamat$ria. What to do .
-IA&.
D. 0. ?' years complains heamat$ria with clotting@ freB$ency@ noct$ria 8 no dys$ria. :iagnosis. .
-Cladder ;/.
D#. What is the most likely ca$se of no changing in varia*ility in fetal heart on (3% Fnon stress
testG .
-0etal sleep
DD. (3% indicated in all e)cept :
-IE74
-:6
--%.
-5! weeks &regnancy
1!!. %he most specific test in E%I in infants is :
-3ample o*tained *y catheter
1!1. -ow do $ calc$late the nat$ral growth of a co$ntry .
-;r$de *irth rate - death rate
1!,. ;hild 5-' years can do all of the following e)cept :
-Imitation
-7ro$p play
-/$tonomy
-/*stract thinking
1!2. /ll of the following are normal d$ring pregnancy e)cept :
-3pider telangiectasia
--eart rate of D!
-:ecreased C&.
-&almar erythema
-White *lood cell 2!!!
(C. %he WC; is increased in pregnant C$t with impaired f$nction
1!5. What is the *est treatment of mild to)emia of pregnancy .
-Ced rest 8 >t. >ateral position
1!'. 0. 1? years old complaining of loss of weight of 1! Kg. /ll of the following s$ggest non organic
ca$se e)cept :
-:oes not worry a*o$t weight loss
-Engaged in athletic activity
-0amily history of eating disorder
-&ersistent complain of fatig$e
(C. Its a case of /nore)ia (ervosa
1!?. Which of the following s$ggest a :6 in a ,! years 6. .
->oss of weight despite good eating
1!. /ll of the following s$ggest thyroto)icosis e)cept :
->oss of weight
-6eanorrghe
-:iarrhea
-3weating
1!#. Oo$ng lady present a feat$re of hypothyroidism Fcold intolerance@ fatig$e@ anore)ia@ weight gain@
meanorraghe@ dry 8 ro$gh skin@ hair dry 8 coarse .... G %3- is increased 8 %5 is low. What is yo$r
treatment .
-3tart >-thyro)ine
1!D. >ady develop severe &&- Fpost part$m heamorrageG@ she present after ? wks with fatig$e an
ina*ility to *reast feed Fa*sence of *reast milkG. /ll will *e decreased e)cept .
-&rolactin
-%3-
->-
-%5
(C. :iagnosis. is 3heehanMs syndrome Fpostpart$m pit$itary infarctG
%he %5 has got a long 1L,t ..
11!. >ady 1' days after delivery complain of epigastric pain radiating to the *ack with na$sea 8
vomiting @ history of gro$p C strep. Infection. %he pain is relieved *y leaning forward@ &E. reveal an
epigastric tenderness what is the diagnosis .
-&ancreatitis
(C. %he pain is relieved *y leaning forward FInglefingerMs signG
111. >ady who delivered after prolonged la*or present to yo$ after , days. /ll of the following
s$ggest Endometritis E)cept :
-%ender $ter$s
-0o$l smelling
-Eter$s D cm. *elow $m*ilic$s
-&yre)ia
11,. >ady after delivery develops fever. 3he still fe*rile after ' days of anti*iotic therapy. &A e)am is
normal. :iagnosis. .
-&elvic-throm*o-phle*itis
112. Oo$ng 6. ;omplains of pain after defecation. &E reveal a *l$e mass @ tender . :iagnosis. .
-E)ternal throm*osed -emorrhoid
-Internal throm*osed -emorrhoid
115. 4ec$rrent perineal a*scess associated commonly with :
-&erianal fist$la
11'. /ll of the following are contraindicated to epid$ral anesthesia e)cept :
-:ecrease platelet
-Cleeding diathesis
-3epsis at site of in1ection
-:ecreased C&.
->a*or at 2' wks.
11?. 6. came after ,5 h. of attack of renal colic. IA& done shows stone !.' cm. at >5@ the kidney is
slightly enlarged. 6anagement .
-O*servation
-%ran$rethral removal of the stone after cystoscopy
(C. ;onservative treatment if the stone is less than ' mm 8 no complication.
11. 6. '' years with painless mass in scrot$m F, cmG transill$mination is FHveG . :iagnosis. .
--ydrocele
-3permatocele
--ernia
-Aaricocele
-%esti$lar ;/.
11#. 6. ,' years presented with solid scrotal mass@ after minor tra$ma to the testis. :iagnosis. .
-3eminoma
11D. E)perimental st$dy done with , dr$gs@ shows that the res$lt is significant@ the epidemiologist
wrote in his report that PpQ is less than !.'. What does that mean .
-%he association is less than 1L,! F'"G likely to *e d$e to chance alone
1,!. Eltraso$nd will diagnose all the following at ,! wks EI;E&% :
-%win pregnant
-&oyhydramnios
-%risomy ,1
1,1. /lfa feto protein is increased in :
-Edward syndrome
-:ownMs syndrome
-6yelomeningocele
1,,. &soriasis ... /ll e)cept :
-:ecreased ;2 @ ;5
1,2. Old man complain of low *ack pain@ I 4ay shows decreased 1oint space 8 osteophytes
diagnosis .
-:egenerative 1oint disease FosteoarthritisG
1,5. Coy , years@ complain of knee pain@ &E tender *$t not swollen knee@ slightly warmer than the
other 1oint. What is the *est investigation to do .
-Iray of the knee
-Cone scan
1,'. &t. present with fract$re@ yo$ want to make a caste. Oo$ advise him a*o$t compartment
syndrome. What will appear first .
-&allor
-;oldness
-($m*ness
-:ecreased p$lse
-&aralysis
1,?. %he most effective treatment in Osteoporosis is :
-E)ercise
-/nalgesics
-Aitamins
-Estrogen
1,. &t. On O;& complain of amenorrhea 2 months after $sing the pills. What is tr$e a*o$t
amenorrhea.
-:$e to high progesterone
-:$e to Estrogen
1,#. :efinition of potential years of life lost F&O>>G : .................................
1,D. 3e)$ally active lady present with dys$ria 8 vaginal discharge. /ll the following can ca$se this
condition EI;E&% :
-7onorrhea
-;hlamydia
-%richomonasis
-;ondyloma /cc$minata
12!. Cacterial vaginosis EI;E&% :
-;l$e cells
-0ishy odo$r with KO- test
-:ecreased lacto*acilli
-Aaginitis
-0rothy offensive discharge
121. &t. complain of vertigo@ how can yo$ diagnose la*yrinthitis .
-(ystagm$s
12,. &t. Operated for pit$itary t$mor@ then he developed poly$ria@ dehydration inspite of receiving 5
> IA fl$id. What will $ give .
-7ive more IA fl$id
-3tart oral rehydration
-7ive vasopressin F::/A&G

(C. %his is a case of 3I/:-
122. /lcoholic &t. ;omplain of insta*ility in gait@ he $ses a cane to avoid freB$ent fall. 4om*erg test
is FHveG. what is yo$r diagnosis .
-:%.
-/lcoholic withdrawal
-/lcoholic cere*ellar syndrome
125. &t. complain of %innit$s in 4t. ear@ on E)am $ fo$nd sensorine$ral deafness. :iagnosis. .
-/c$stic ne$roma
12'. (ew *orn@ all the following are increasing risk factors of deafness EI;E&%:
-0amily history
-Esed of gentamycin
--istory of ;6A infection
-:elayed speaking
12?. What will *e yo$r management. 0or a 2 years *oy with st$ttering .
-4eass$rance
12. /ll the following neonates are prone to hypoglycemia EI;E&%:
-IE74
-:ia*etic mother
-(ormal infant *orn at 2? wks
--ypothermia
-(on of the a*ove
12#. 2, years 0. cons$lt $ for ina*ility to conceive for , years@ she is married who has 2 children from
another woman. %emperat$re chart shows ov$lation. What to do first .
-C--;7
--ysterosalpingogram
-&rogesterone at ,1 day
-3emen analysis
12D. 0. 1# years height 1'# cm @ normal *reast@ with primary amenorrhea @ &E small $ter$s@ all of
the following to *e done EI;E&% :
-%0% F%hyroidG 8 &rolactin level
-&rogesterone test
-03-@ >- level
->aparoscopy
-3k$ll I 4ay
(C. 6;;R ,!!1 F7O 11G we normally do the others in the same order as it is ..
15!. 0. ,1 years @ with , months history of fi*roid mass@ asymptomatic. 6anagement .
-4eassess after one year
-6yomectomy
--ysterectomy
--ormonal therapy
151. What is the most common so$rce of merc$ry poisoning in ;anada . 8 who are the most e)posed
gro$p of people in ;anada .
-0ish - Fnative IndiansG
15,. Which of the following yo$ sho$ld not notify the police a*o$t .
-;hild a*$se
-3po$se a*$se
-;hild neglect
-(one of the a*ove
152. Oo$ do a stress test E;7 for a &t. /ll are significant EI;E&% :
-3% decreased *y more than , mm
-;hest pain
-3% increased F$pward sloppingG
-:ecreased in C& *y more than '! mm-g.
155. /ll are tr$e with post infectio$s 7(. E)cept :
-(ormal ;2 @ ;5
--ypoal*$minemia
-Increased cholesterol
-&rotein$ria is more than , gm.
15'. 6. ?' years @ present with low *ack pain anemia 8 hypocalcemia. What will yo$ do to confirm
the diagnosis .
-WC;. ;o$nt
-Cone marrow aspiration
-&latelet co$nt
15?. -IA &t. With fever 8 weight loss @ he may have all of the following EI;E&% :
-Esophageal candidosis
-6yco*acteri$m /vi$m
-6ycoplasma pne$monia
-;6A infection
-6ycoplasma cell$lari
15. 0. 2' years with ata)ia@ weakness of the >t. 3ide of *ody @ dysphagia 8 vis$al tro$*le . &E
reveals diplopia 8 nystagm$s. What is the :iagnosis. .
-6$ltple 3clerosis.
15#. What is the most common complication of N$venile 4he$matoid /rthritis:
-Iridocyclitis
--epatitis
-(ephritis
-vasc$litis
15D. 6. Cro$ght to $ *y police@ for assa$lting a 1, years old *oy. When asked his name *y the
police @ he said that he donMt know @ 8 when $ ask him how many legs does the horse have @ he said '
. What is the diagnosis .
-6alingering
-0actitio$s disorder
-/ntisocial personality
-0$g$e
1'!. In which pathology yo$ have gloves 8 stocking anesthesia .
-:6.
1'1. 6. With *ipolar disorder came with agitation 8 aggressive *ehavior. Cest treatment .
-IA -aloperidol
->ithi$m 8 /ntipsychotics
1',. /ll are seen in :%. E)cept :
--%.
-0ever
-%achycardia
-0i)ed hall$cination
1'2. Cest prophyla)is for a ;anadian traveling to a 6alaria area with no resistant train is :
-;hloroB$ine
1'5. &t. :iagnosed with dysthymia what is the *est treatment :
-&sychotherapy
-0l$o)etine
-Imipramine
1''. In cardiac arrest d$e to Aentric$lar 0i*rillation @ what is tr$e .
-(a-;O2 sho$ld *e given every '-1! min$tes $nless *lood gas are known
->ignocaine m$st *e given
-:; cardioversion first
1'?. :yspepsia@ all are helpf$l EI;E&%:
-/ntacids
-Elevate *ed head
-;imetidine
-6isoprostol
1'. /sthmatic &t. On steroids @ has to go for operation for perforated d$odenal $lcer . What will $
do .
-1!!-,!! mg -ydrocortisone IA *efore the s$rgery
-Ceta-agonist inhalor *efore the s$rgery
1'#. ;hinese resta$rant food poisoning. /ll are tr$e EI;E&% :
-3ymptoms within 1L, h.
-4esolves after ,5 h
-3ymptoms of ashma H ata)ia H dyspnea
(C. 6onosodi$m gl$tamate is the ca$se of this syndrome Fits a pharmacological *$t not allergic
reaction. :ose related symptom of *$rning sensation thro$gho$t the *ody@ facial press$re@ an)iety 8
chest painG
1'D. 4/. &t. On aspirin present with a normocytic normochromic anemia . /ll are tr$e e)cept:
-4espond to treatment with C.1,
-0eritine may*e Increased
-:ecreased ser$m Iron
-(ormal total Iron *inding capacity
1?!. What is the most common ca$se of congenital heart fail$re in (C.
-A3:
-/3:
-%7/
-%etralogy of 0allot
1?1. / n$rse with hypoglycemia 8 decreased ;-peptide. What to do .
-4efer to &sychiatrist
1?,. /ll are feat$re of %33 E)cept :
-0ever
-:ecreased C&.
-&$st$lar skin lesion
-Erythroderma
1?2. /ll are feat$re of K/W/3/KI disease EI;E&%:
-0ever for ' days
-&$r$lent con1$nctivitis
-3kin rash
-0iss$re of lip
-3traw*erry tong$e
1?5. Aaria*le deceleration commonly seen in :
-Eteroplacental Ins$fficiency
-;ord compression
-3leeping fet$s
1?'. /ll increase the level of car*ama9epin EI;E&% :
-;imetidine
-Estrogen
-&heno*ar*ital
1??. 6. &resent with 4t. 3ide hornerMs syndrome 8 contralateral loss of sensation of the *ody.
Where is the site of the lesion .
-&I;/
-I;/
-Internal ;arotid
1?. &t. With hypokalemia . /ll tr$e EI;E&% :
-:iarrhea
-&aralytic ile$s
--ypotension
--ypoventilation
-6$sc$lar weakness
-&oly$ria 8 polydepsia
1?#. /ll are live atten$ated vaccine e)cept :
-C;7.
-Aaccine of yellow fever
-&olio
-6easles
-:iphtheria
1?D. -ow often do $ give a *ooster in people previo$sly adeB$ately imm$ni9ed against tetan$s :
-Every 1! years
1!. /ll are incl$ded in the ro$tine imm$ni9ation EI;E&%:
-6easles
-&neomococ$s vaccination
-6$mps
-4$*ella
11. -eamophil$s Infl$en9a contact@ prophyla)is is :
-4efampicine
1,. &regnant @ # wks@ came into contact with a *oy s$spected of having 4$*ella infection .
6anagement .
-4$*ella vaccination
-7ive 4$*ella Ig7.
-:o anti*ody titre in the pregnant lady
-%herape$tic a*ortion
12. 0armer came with a nail in his foot 8 temp. of 2#., . 6anagement :
-/nti*iotics
-%etan$s to)oids H antititan$s Ig. H anti*iotic
-/ntitetan$s Ig.
15. 0. Oo$ng present with 6eanorrhagea. ;ommonest ca$se is :
-:eficient l$teal phase
-3hort secretory phase
-&rolonged secretotey phase
(C. /nov$lation is the *est answer if there.
1'. &ostmenopa$sal lady with 6I. 8 she is an active smoker. 3he does not want to stop smoking.
3he is on -ormonal 4eplacement %herapy. What advice do $ give .
-3top Estrogen 8 start e)ercise
1?. &ost-Op contin$ed to *leed despite 1! $nits of packed 4C;. %ransf$sion. What is the ca$se .
--ypercalcemia
--yperkalemia
-:il$tional throm*ocytopenia
1. 4/. &t. On /3/. :eveloped I%& . What is the treatment .
-3teroids
1#. &t. :eveloped e)cessive menstr$al *leeding 8 p$rp$ra . /ll the following la*. 0inding are
e)pected EI;E&%:
-:ecreased -*.
-:ecreased platelets
-Increased platelets
-&rolonged *leeding time
1D. Cartholinitis a*scess. What is the treatment .
-/nti*iotics
-6ars$piali9ation
(C. I 8 : is a good answer as well.
1#!. &t. With lymphadenopathy 8 splenomegaly 8 raised WC; . What is the confirmatorey test .
-;C; deferential
-Cone marrow aspiration
(C. >e$kemia case
1#1. &t. With *ilateral hilar lymphadenopathy with micronod$lar infiltrates. /ll can give this feat$re
EI;E&% .
-3arcoidosis
-%C.
-%rans*r*ronchial *ronchogenic carcinoma
1#,. &t. With painf$l *l$ish mass protr$ding from the an$s. :iagnosis. .
-&rolapsed anal piles
-%hrom*osed e)ternal piles
-Internal piles
1#2. /ll can ;a$se &r$ritis ani EI;E&%:
->a)ative
--emorrhoids
-0ecal incontinence
-;olorectal ;/
1#5. 0. ,5 years with FHveG preg test @ present with , days vaginal *leeding@ E3 shows 2 cm mass in
the >t. /dne)a 8 empty $ter$s@ pregnant 3ympt 8 signs of preg has disapeared 8 the cervi) is
closed. :iagnosis. .
-;omplete a*ortion H l$teal cyst.
(C. $ consider ectopic pregnancy $ntil otherwise proven.
1#'. Indication of ;-section in *reech presentation is:
-0ootling
-0ront*reech presentation
1#?. &revalence definition: total n$m*er of cases in a pop$lation at a given time.
life *irth
1#. ;r$de *irth rate < ------------------------------- ) 1!!!
%otal medial pop$lation
1##. %he tree of a$tosomal dominant transmission
1#D. 0. ;ons$lt yo$ *eca$se of a possi*ility of her offsprings FkidsG having -$ntingtonMs ;horea. -er
father is affected. -er h$s*and family donMt have this disease What is the possi*ility that her
children will develop the disease .
-'! "
-! "
-,' "
-1!! "
1D!. 0. With pregnancy like symptoms@ E3 shows grape like masses in the $ter$s. /ll tr$e EI;E&%:
6$ltiple gestation
-Cilat. Ovarian cyst
--yperthyroidism
--ypothyroidism@ coma@ preor*ital my)edema
1D1. /ll have risk factor to develop preeclampsia:
-($lipara
-:6
--istory of preeclampsia
-6$ltiparity
1D,. &regnant &t. ,! weeks %he $ter$s si9e is ,' cm. /ll tr$e EI;E&%:
-(ormal pregnant
-Wrong estimation of gestational age
-%win pregnant
-Cilat. 4enal dysgenesis
1D2. Old lady with ankle edema at the right medial malle$l$s@ with s$perficial $lcer 8 s$rro$nding
scar. :iagnosis. .
-Aen$s ins$fficiency
-Aen$s valv$lar ins$fficiency
1D5. &reg. >ady with known fi*roid@ complaining from a*dominal pain
-4ed degeneration
1D'. 6. 1? years old complain from loss of weight@ he is passing *loody diarrhea@ Investigation show
normal $pper indoscopy 8 segmoidoscopy. What to do ne)t .
-Epper 7I series
1D?. ;hild 5 years old@ presented with post. 6ediastinal t$mor. What is the commonest germ cell
t$mor .
-%hymoma
-(e$ro*lastoma
-lymphoma
1D. &%. With fat mala*sorption@ passing floating stools H loss of weight 8 easy *r$ising. :iagnosis. .
-7I lymphoma
-7I %C
-7l$tane antheropathy Fceliac diseaseG
-4epeated episode of pancreatitis
1D#. &t. With :K/. /ll occ$r EI;E&%:
-Increased /mylase
-/*d. &ain
-Increased K.
-Increased osmolarity
->e$kocytosis
(C. 6erck 1DDD Fp.1#G in :K/. ser$m amylase typically elevated while K is Increased or normal
1DD. 0. &t. On Ins$lin therapy@ she has a *a*y. Oo$ will s$ggest her the following a*o$t Ins$lin
E)cept:
--old ins$lin $ntil the *a*y start eating
(C. Falways Ins$lin 8 -eparin are 3/0E d$ring pregnancy 8 lactationG
,!!. Cest site to o*tain smear for 7onorrhea in a woman is:
-A$lva
-Aagina
-;ervi) Fmo$th of the wo$ndG
,!1. Which stage of se)$al e)citement according to %onom criteria when the female is having clitoral
erection@ increased vaginal secretion@ nipple erection .
-Early e)citement
-4esol$tion
->ate e)citement
->atent phase
,!,. 0. With diffic$lty caring shopping *ag@ weak a*d$ction of the th$m* H minor wasting of the
th$m* Fminor thenar atrophyG :iagnosis. .
-;arpal t$nnel syndrome
,!2. 4adial nerve in1$ry at the level of the head of 4adi$s. What will $ find .
-:ropping wrist
(C. 6erck 1DDD Fp.15D2G %he 3at$rday night palsy is another name for the 4adial nerve palsy.
,!5. Which of the following is a se)$al desire a*normality
-:oes not imagine se)$al fantasies
-(o se)$al activity for more than , wks
-(ervo$s a*o$t se)$al act
-%ry to avoid partner contact
,!'. %ransvestitism disorder@ all tr$e EI;E&%
-:ressing clothes of other se) for se)$al e)citement
-6ore common in male
-%hey are female
,!?. /ll are known to ca$se depression EI;E&%:
-;lonidine
-;imetidine
-;yclosporine
(C. %he dr$gs who ca$se depression are : /nti--%@ /nti-&arkinsonian@ -ormones@ 3teroids@ /nti-
%C@ /nti-(eoplastic
,!. &t. Cro$ght to $ m$te 8 inaccessi*le to e)amination. :iagnosis. .
-;atatonia 6;;R &3? F,!!1G.
,!#. 6. 2' years 0o$nd in the street wandering@ he doesn't know his name@ talk to him self. /ll are
possi*le etiology EI;E&%:
-3chi9ophrenia
-0$g$e
-/l9heimer disease
,!D. 7irl came with midline neck cyst@ which moves while protr$ding the tong$e Fwith swallowingG.
:iagnosis. .
-:ermoid cyst
-Crachial cyst
-%hyroglossal d$ct cyst
,1!. /ll are ro$tine investigation for an old lady with $rinary incontinence EI;E&%:
-Erine analysis
-;ystogram
-Erinary diary
-:irect vis$ali9ation of incontinence when &t. with f$ll *ladder while co$ghing
(C. (ever let her pee on yo$ :G
,11. :efinitive test for chronic pancreatitis is
-;%
-E4;&
-64I
,1,. &t. With pr$ritis 8 progressive 1a$ndice@ with increased />&. 8 total *ilir$*ine. What is the
ne)t investigation $ do .
-E3.
-&%; Fperc$taneo$s tranhepatic cholangiographyG.
-E4;&
(C. /lways start with the less invasive investigation.
,12. In (ecroti9ing Enterocolitis. /ll feat$re e)ists EI;E&%:
-&ne$matosis Intestinalis FI 4ayG
-&eritonitis
-&eritoneal calcification
-&ortal veno$s gas FI 4ayG
-&ne$mo peritoni$m FI 4ayG
,15. What is the commonest ca$se of non scarring alopecia on a yo$ng girl .
-/lopecia /reata
-%enia ;apitis
-;hemotherapy
,1'. :$ring insertion of IE:. %he &t. :evelop hypotension. What will $ do .
-Elevation of the legs
,1?. IE:. %read seen dangling o$tside the cervical os d$ring an e)amination of a pregnant &t. What
will happen if removed
-(ormal pregnancy ''"
,1. &t. In the E4. With fract$re of the fem$r @ develop s$dden hypotention. What is the ca$se .
-hemorrhage
,1#. Aerapamile given IA for treatment of 3A%. %he &t. :evelop hypotention. What will $ do.
-:efi*rillation Felictrical cardioversionG
,1D. &t. With psychostim$lant into)ication. What is the *est management. When the &t. Will *e
discharged .
-Cen9odia9epines
-:aily follow $p in s$pport gro$p
,,!. /ll occ$r in :%. EI;E&%:
-%remor
-Ais$al hall$cination
-:el$sion
-&sychomotor activity
,,1. /*o$t contact dermatitis. /ll tr$e EI;E&%:
-6ostly with plant e)pos$re
-3ystemic steroids co$ld *e $sed
->ocal steroids are not $sef$l
-3kin test is $sef$l
,,,. /ll can ca$se inverted $ter$s EI;E&%:
-IA o)ytocin stim$lation
-/tonic $ter$s
-&ress$re on soft f$nd$s
-&$ll cord forcef$lly
,,2. Who is responsi*le for the $ltimate FstandardG health care B$ality .
-Cord of directors
-3taff
-;hief physician
-;hief of staff
,,5. What is the r$le of 4oyal collage of ;anada .
-;redentialing *ody Fcertification of specialistG
,,'. &regnant &t. With *icorn$ate $ter$s. What is the likely o$tcome of pregnant .
-/*ortion
-&ret$rm delivery
-(ormal pregnancy o$tcome
,,?. &t. With plasmodi$m viva) malaria@ treated 8 rec$rrence. 6ost likely ca$se .
-(o Eradication of liver forms
,,. %he *est screening for hyperaldosteronism in -%. Fconn diseaseisG:
-3er$m renin activity level
(C. 4enin will *e increased in secondary 8 decreased in primary
,,#. /3/ $se H alcohol a*$se over weekend@ the pt. :eveloped 7I *leeding. 7astroscopy shows
m$ltiple areas of erosions. /ll can *e done EI;E&%:
-(7 t$*e
--, *locker
-7astrectomy
-Aasopressin
,,D. 0. &t with FHveG /(/@ presented with ankle edema 8 arthralgia@ protein H in $rine. :iagnosis. .
-3>E
-3cleroderma
,2!. :iagnosis. Of ;ondyloma /c$minata in pregnancy. Oo$ give all EI;E&%:
-&odophyllin
,21. Elderly &t. : &refer to sit home 8 to *e visited *y relatives.
,2,. In Wolf &arkinson White syndrome : shortening of &4 interval 8 :elta wave.
,22. ;hild complain of headache@ a*dominal pain@ na$sea@ vomiting then he sleeps. :iagnosis. .
-/*d. 6igraine
,25. Ait. K. :eficiency in new *orn will res$lt into :
-Increased &%. 8 &%%.
(C. Ait. K. F6enadioneG %o)icity will present with hemolytic anemia 8 kernikter$s
,2'. Which one of the following affect the o$tcome 8 feat$re planning of ;anadian health
program .
-/ccident
-/ge
,2?. &ost-part$m psychosis EI;E&%:
-;an ca$se resid$al effect
,2. Cest treatment for shigellosis is:
-3eptrim Fcotrimo)a9olG
,2#. &hoto of down's syndrome *a*y. What will confirm the diagnosis .
-Kariotypes
-%otal *lood co$nt
,2D. Epileptic &regnant &t. ;ontrolled on phenitoin asked for yo$r advice a*o$t contin$ing the dr$g
which she $se .
-the *enefit of risk o$t weight of it's teratogenicity F%o contin$e $sing itG
,5!. Cest screening test for hypothyroidisim :
-%3-
,51. 6an had fighting pro*lem at work. (ow he complains from *ack pain. &E shows no
pathological finding *$t the man is depressed 8 o*sessed with it. :iagnosis. .
-6alingering
--ypochodriasis
-3omatoform pain disorder
,5,. ;harming manip$lative does not learn from e)perience with lack of remorse Fg$iltG. What &: is
it:
-/ntisocial &:
-Corder line &:
--istrionic
-(arcissistic
(C. %he key is lack of remorse Fg$iltG.
,52. What is hypochondriasis .
-Its a *elief of having chronic disease despite good reass$rance
(C. %he &t. is sta*le 8 has no social pro*lem 8 he will visit many doctors for the same pro*lem@ he
recogni9es that his reaction is e)aggerated
,55. 6other *ro$ght her *a*y to $ @ he is non sensitive to other feelings @ having pro*lem with
learning 8 speech. :iagnosis. .
-/$tism
-3pecific learning disorder
,5'. /:-:. What is tr$e .
-&ersistent to ad$lthood
-/ntisocial
-Increase criminal rate
,5?. Which are associated with highest risk of s$dden death:
-6$ltiple Aentric$lar /rrhythmia on ,5 h. -olter monitor
-E1ection fraction less than ,! "
-0amily history of s$dden death
-/trial fi*rilation on aprevio$s /6I
-3evere narrowing of more than 1 coronary art.
,5. 4egarding s$icide all are increased risk of s$ccessf$l attempt EI;E&%:
-:epression
-6alignancy
-0emale gender
->onely white old man
,5#. 6ost imp. :ietary modification of middle aged gro$p is :
-%otal calorie content
-0i*er diet
-;-O content
-protein
,5D. In 63. /ll commonly occ$r EI;E&%:
-:iplopia
-(ystagm$s
-3cotoma
-/nosmia
-%remor
,'!. %reatment or dr$g of choice for cyclothymia
->ithi$m
-&sychotherapy
--aloperidol
,'1. In C(. /ll occ$r EI;E&%:
-:ental caries
-7I refl$)
-&arotid enlargement
-3tomach r$pt$re
-6eta*olic acidosis
,',. 3tridor is present in all EI;E&%:
-4etropharengeal a*scess
-/c$te asthmatic attack
-%racheolaryngo*ronchitis
-/c$te Epiglotitis
,'2. &t. With dia9epam withdrawal. Which one can't $ give him .
-;hlordia9epo)ide
-:ia9epam
->ora9epam
-C$spirone
,'5. /ll are meas$re to avoid Cot$linism@ EI;E&%:
-/ntiacid
-Celow free9ing temperat$re
-Aac$$m
-(itric preservation
,''. :A% in postpart$m period lady. /ll can *e given EI;E&%:
-;o$madin
--eparin
-Early mo*ili9ation
(C. We can $se it to avoid the :A% 8 not after...
,'?. Which is tr$e a*o$t mastitis .
-7ram F-veG commonest
-I8: is the treatment
-/ssociated with lactation
,'. What is the commonest ca$se of pse$domem*rano$s colitis .
-;lostridi$m dificile to)in
,'#. 4elation *etween as*estosis 8 smoking is called :
-/dditive
-3ynergistic
-antagonist
,'D. :efense mechanism in pho*ia is :
-/voidance
-splitting
,?!. :efense mechanism is *order line &:. is :
-/voidance
-3plitting
,?1. 4t. -anded lady complains of feeling of *eing dissociated from environment@ e)pressive
dysphasia 8 a$tomatism. Where is the lesion .
->t. %emporal
-4t. %emporal
->t. &arietal
-4t. &arietal
-0rontal
,?,. Coy fell do$n from a tree@ he started a*dominal &ain 8 heamt$ria. What will *e yo$r ne)t
investigation .
-4enal scan
-IA&
-4etrograde $rography
-3$rgical e)ploration
,?2. >ady complains from *ilateral a*dominal >ower R$adrant pain with tenderness after the end
of cycle *y ,-2 days. What is the diagnosis .
-Endomettriosis
-3alpingitis
,?5. What is the commonest ca$se of intestinal o*str$ction in males over ?' years. .
-posts$rgical adhesion
,?'. ;hild swallow a ,.' cm screw@ I 4ay shows it in the $pper B$adrant region. What will *e the
management. .
-3$rgery
-4eass$re 8 follow $p Irays after few days
-Endoscopy
,??. &remat$re r$pt$re of mem*rane associated with all EI;E&%:
-&ne$monitis
-3eptisemia
-4:3
,?. 7onorrhea treated with spectinomicin @ after there is rec$rrence of symptoms. What is the
diagnosis 8 what will *e the management. .
-;hlamydia 8 give tetracycline
,?#. What is the diagnostic test for myasthenia gravis .
-%ensilon test
,?D. What is the diagnostic test for c$shing syndrome .
-:3% Fde)amethasone s$pression testG
,!. -ypertensive &t. &E. shows *ilateral 0emoral *r$its. :iagnosis. .
-4enal stenosis
,1. 0. &t. ;omplains from severe headache@ sho$lder pain 8 *ilateral Wrist weakness. :iagnosis. .
-4/.
-%emporat /rteritis
-&olymyositis
-3arcoidosis
,,. 6a)im$m time fro primary syphilis to *e seen after se)$al contact is :
-%hree months
,2. /ll are feat$res of se)$al aro$sal EI;E&%:
-%$mescence
-&elvic congestion
-Eterine contraction
,5. &t. -as lower lim* lymphedema following pelvic radiotherapy. What is yo$r management. .
-/3/ reg$larly H di$retics occasionally
-;ompression stocking H raising leg at *ed time
,'. ;hild with *ilateral C$rns of his lower lim*@ his mother said that crawling near the heater.
What is the diagnosis .
-3haken *a*y syndrome
-;hild a*$se
,?. &t. With history of -%. 8 :6. @ get ac$te 6I. 8 died d$e to cardiac arrest. In writing the death
certificate@ what is the immediate ca$se of death. .
-:6.
--%.
-6I.
-;ardiac arrest
,. &t. ;omplains of painf$l swelling on the face with fever 8 malaise. &E: erythemato$s swelling
on the cheeks 8 nose. What do $ e)pect to find on e)amination .
-Aesicles on the ipsilateral ear
-Well demarcated sharp margin
(C. %he diagnosis is Erysipelas
,#. / woman came to yo$r clinic claming that she is seeing creat$res form the space@ trying to kill
her. -er h$s*and denies the story. 3he was $ncooperative. What sho$ld yo$ do .
-/dmit 8 start antipsychotic treatment
-3end her home@ ask h$s*and to p$t her treatment in coffee 8 give it to her
(C. :iagnosis. is schi9ophrenia
,D. %hia9ide di$retics are the first choice for:
--% alcoholic with maln$trition
--% H :6
--% H go$t
--% in elderly &t.
--% H >A0
,#!. Oo$ng 0. &t. with acne on face. &E: open 8 closed comedones H , pap$les on chin. What is the
treatment .
-Cen9oyl pero)id
-Oral tetracyclin
-steroids
(C. :iagnosis. Is /cne v$lgaris
,#1. ;hild present with severe dehydration with hypotension@ investigation shows:
(a 1,! mmol.L>.
K 2.! mmolL>.
CE( 1'mmolL>.
Oo$ decided to rehydrate orally. What will yo$ give .
-,! ml. (a '!" H K 1!"
-'!" normal saline H gl$cose '!" H ,! mmol K;l.
-&$re Water
-'" s$gar in water
-milk.
,#,. 6. ,' years. -istoty of diarrhea for 1' days@ he has lost ' kg.@ *ari$m enema 8
rectosigmoidoscopey were normal. What is yo$r ne)t management. .
-3tool c$lt$re
-3tool microscopy
-&lain a*dominal I 4ay
-Epper 7I series
,#2. What is tr$e 4egarding smoking restriction in p$*lic places.
-:ecreases the incidence of chronic respiratory disease
-:ecreases the incidence of allergic respiratory disease
-:ecreases the incidence of cardio vasc$lar disease
-3hortens hospital stay
,#5. 4egarding conversion disorder. /ll are tr$e EI;E&%:
-/phasia
->a *elle indifference
-Invol$ntary character of the syndrome
-;an ca$se social or professional hindrance
-;an *e ind$ced vol$ntarily
,#'. / new vaccine against measles is *eing eval$ated. %he following res$lts of one f$ll year of
st$dy :
:iseased -ealthy %otal " age
nonvaccinated 12 --------- 2# 2?.2!
Aaccinated less 1 yr '1 --------- ,?? 1.,#
Aaccinated more 1 yr ----- --------- 5!# --------
What is the Aaccine efficacy for the vaccinated *efore one year
- !.? "
-1.D "
-5D.! "
-,1.# "
-2?.2 "
(C. %he efficacy < incidence of non-vac - vac
,#?. In a small town. %he incidence of disease / is Fi/G #L1!!! . %he disease C not related to disease /
has an incidence of FiCG 1,L1!!! . What is the pro*a*ility that the &t will have *oth of the disease In
the same time .
-i/ H iC
-i/ I iC
-Fi/ H iCG L 1 - Fi/ H iCG
-Fi/ H iCG L ,
-Fi/ H iCG L Fi/ - iCG
,#. %he following disease are more freB$ent in poor than in rich ;anadians EI;E&%:
-3$icide
-;oronary art. disease
->$ng ;/
-3troke
-Creast ;/
,##. %he mother of a 1, years old child has to *e admitted to the hospital for , wks. %he *oy present
with loss of appetite@ he remains in his *edroom 8 has insomia@ his stool freB$ency is red$ced. &E:
soft a*dominal 8 all the rest of the e)am is normal. :iagnosis. .
-7I infection
-Early schi9ophrenia
-0l$
-3eparation an)iety
-neglect
,#D. ;hild 1! years with pyre)ia 8 N4/ Fstill's diseaseG What is the most common complication.
-Endocarditis
-Knee 8 hip arthritis
-/septic meningitis
-Iridocyclitis
-(ephrotic syndrome
,D!. Which of the following vaccine ca$ses more side effect :
-&ert$ssis
-6easles
-:iphtheria
Oellow fever
,D1. Of the following ;/. Which one is ca$sed *y vinylchloride :
->$ngs
-Esophag$s
-4hinopharyn)
->iver
-Cladder
,D,. 0. 2 years has pro*lems at work for several months@ she also has episodes of hyperactivity 8
e$phoria. %hese were preceded *y episodes of sadness 8 ina*ility to cooperates with her colleag$es
inspite trying hard to do her *est. :iagnosis. .
-Cipolar disorder
-:ysthymia
-;yclothimia
-6asked depression
-0actitio$s disorder
,D2. / new test for ;/ ;ervi) allows *etter detection. 3tatistically all can *e modified EI;E&%:
-&revalence
-Incidence
-&&A
-(&A
-3$rvival rate
,D5. &ropranolol can *e $sed in all EI;E&%:
->ithi$m ind$ced tremor
-&anic attack
--yperglycemia
--%.
-/ngina
,D'. (ew*orn , wks old *ro$ght *y mother@ she complains of decreased milk secretion. %he *a*y
weights 5.D kg. %he *irth weight was 2. kg. %his pro*lem is partic$larly marked in the evening.
What is the most appropriate treatment
-&rolactine , mg. Cid to the mother
-3$pplement the *a*y with one spoon of solid food every evening
-3$pplement with form$la every evening
-4eass$re the mother
-(o *reast feeding in the evening to allow more milk to the ne)t morning
,D?. 0. 1 years with severe menorrhagea. Investigation reveals -* #.! gmLd>. &$lse 11! C&. #!L?!.
Ch;7 F-veG. Initial management. .
-IA O)ytocin
-Clood transf$sion
-:8;
-I6. 6edro)yprogesteron
-IA. Estrogen
,D. ;hild with *ronchospasm after hockey game. /ll can *e given EI;E&%:
-/minophylin
-3al*$tamol
-;romoglycate
-Cetamethasone
-O)ygen Ffacial maskG
,D#. &t. ?' years with ;O&: treated with o)ygen F0iO, 55"G present with the following test res$lts:
&aO, &a;O, p-.
?'" 2'" .2!
'," 5#" .,?
5#" '," .,1
What is management. .
-O, 1!!"
-Increase 0iO, to ?'"
-3top O,
-Int$*ate
-IA /minophylin
,DD. %he following are carcinogenic EI;E&%:
-0at
-6onosodi$m gl$tamate
-Cen9ene
-(ickel
-arsenic
2!!. ;hild 1# m. Old with dyspnea. I 4ay shows normal 4t. >$ng@ >t. >$ng distention.
mediastin$m deviated to the 4t. With transl$cent >t. l$ng. :iagnosis. .
-43A infection
-&ne$monia
-0oreign *ody
-;ongenital anomaly
-;ongenital p$lmonary aplasia
2!1. /ll are carcinogenic EI;E&%:
-Erani$m
-/rsenic
-4adon
-;hromi$m
->ead
2!,. Which of the following radiation has the deepest penetration in h$man tiss$e:
-/lpha
-7amma
-Iray
-:elta
-EA
2!2. Which is tr$e regarding the side effects of radiation .
-6icrowaves ca$se s$perficial *$rns
->aser radiation can ca$se retinal *$rns
-EA can ca$se /lopecia /reata
-Ceta ray can ca$se >e$kemia
-:elta ray are dangero$s for the central grey n$cle$s
2!5. ;haracteristic of place*o effect. /ll tr$e EI;E&%:
-'!" effective
-&t. Celieves in treatment efficacy
-Its greater in I6 than in &O.
-It decreases if $sed freB$ently *eca$se of dependence
-(alo)one effect shows that place*o effect occ$rs *y inhi*ition *y morphinic receptors
2!'. /ll are tr$e a*o$t %$rner syndrome EI;E&%:
-3hort stat$re
-/mmenorrhea
-;oarctation of the /orta
--irs$tism
-/cne 8 micrognatia
->ow set hair
2!?. /ll are ro$tinely done in infertility investigation EI;E&%:
-3emen analysis
-%emperat$re chart
-03-. >-.
--37
-&ost coital test
D
2!. 0. 5 years old admitted with $rinary catheter@ she has E%I. Erinanalysis shows 1! L high
power field. ;$lt$re shows sensitivity to nitrof$rantoine@ %6&L36I@ gentamycin@ tetracyclin. &E:
asymptomatic 8 no fever. What is yo$r management. .
-(itrof$rantoin 2 ta*.Lday for ,! days
-7entamycin , in1ection I6. L day for 1! days
-%6&L36I 1 ta*.Lday for 15 days
-:o)ycyclin 5 ta*.Lday for 1! days
-(o treatment
2!#. In which case occ$pational deafness is mostly s$spected
-Ena*le to hear the *ell so$nd at 2 m.
-:ecreased in hearing high intensity voices
-:ecreased peak at 5!!! hert9 on the a$diogram
-:eafness at speaking voice

2!D. %he difference *etween 6. 8 0. 3e)$ality is:
-0. -ave more inhi*ition
-0. -ave more erogeno$s 9one
-0. Orgasm is less intense
-0. 4efractory period is shorter
-0. Orgasm is shorter
21!. &ict$re of old 0. With temporal arthritis asking a*o$t which is common in these &t. :
-Naw cla$dication
--eadache
-*lindness
211. &ict$re of a lady with goittrons pap$les 8 pro)imal m$scle weakness. :iagnosis. .
-:ermatomyositis
-6i)ed ;onnective %iss$e disease
-3cleroderma
21,. &ict$re of a lady genitalia showing vesicles aro$nd v$lva. :iagnosis. .
-3yphilis
-;omdiloma ac$minata
--erpes simple) genitalia
212 pict$re of cl$**ing. Which is not ca$sing it .
-;hrons's disease
-;hronic *ronchitis
-;iliac disease
215. / child with fever @ irrita*ility 8 drooling. &E: there are n$mero$s small $lcers on tong$e H ant.
;ervical >ymph node enlargement :
--erpangina
--erpes simple) stomatitis
-le$coplakia
21'. &t. With schi9ophrenia 8 on treatment. With akathisia Fhe cant sit stillG What is the *est
treatment is .
->ora9epam
21?. Which of the following dr$gs is the safest is &t. With depression 8 heart pro*lem .
-0l$vo)amine
-Imipramine
-:o)epine
-:espiramine
-&henel9ine
21. &t. With fatig$e 8 no pleas$re 8 decrease sleep. :iagnosis. .
-:epression
-/n)iety
-Cipolar
21#. &t with :6. 8 -%. On propranolol@ complains of progressive impotence 8 he didn't have
se)$al interco$rse with his wife for more than ? months -e came to $ with his wife@ she told $
privately that he has morning erection specially when he see ad$lt maga9ine. What is yo$r reaction .
-:o night t$mescence test F4E6 testG
-6arital psychotherapy
-3top propranolol
-Impotence d$e to :6. Fvasc$lar pro*lemG
21D. Cest treatment for :%.
->ora9epam
-
-MSTMSTMSTMSTMSTMSTMSTMSTMSTMSTMSTMSTMSTMSTMUVWWWWXSTPYMUVWWWWXSTQYMUVWWWWXSTPYMUVWWW
WXSTQYMUVWWWWXSTPYMUVWWWWXSTQYMUVWWWWXSTPYMUVWWWWXSTQY
2,2. &regnant d$ring delivery Ff$ll termG@ cephalic-occip$toposterior@ head of the *a*y at station F-
1G &t. Is e)ha$sted 8 takes long time in ,nd stage of la*or. What is yo$r management.
-3ymsons
-K1elands forceps
-Aac$$m delivery
-;-section
2,5. &t with retain placenta. /ll are associated EI;E&%:
-Aaginal *leed
-Infection FendometritisG
-&$rp$ra aro$nd $m*ilic$s
2,'. ;hild with microcytic anemia with (a$sea@ vomiting@ anore)ea 8 costipation also has sleep
dist$r*ances 8 irrita*ility. :iagnosis. .
--g. &oisoning
-%halassemia
-C&; poisoning
->ead poisoning
-Ait. / poisoning
2,?. 6ost common so$rce of silicosis e)pos$re in ;anada is:
-3and*lasting
-;oal d$st
-Crake lining
--igh consentration of 7ases
2,. Which of the following is the most common trigger of /sthma in children .
-:$st ho$sing
-4$gs
-&arental smoke
-&ets
2,#. What is the mode of transmission of -ydrocephal$s .
-%risomy
-6onosomy
-E$ploidy
2,D. %he most ;ommon ca$se of non reactive (3% Fnon stress testG is :
-3leeping fet$s
-6$ltiple gestation
-Intra$terine death
-6orphine
22!. ;ommonest ca$se of hirs$tism in reprod$ctive age gro$p:
-;onstit$tional
-&olycystic Ovary disease
-;ongenital /drenal -yperplasia
221. Ediopathic heamat$ria with no other a*normality. 6anagement .
-&rednisolon
-0ollow $p the &t. /fter one year
->EE&
22,. Which of the following is tr$e regarding ;hromoglycate .
-;ontraindicated with steroids
-(ecessary in ac$te attack of asthma
-&revent *inding of IgE with cells
-&revents histamin from synthesi9ed cells
222. 6. 5, years with gl$cos$ia@ dark skin@ palpa*le liver. Which will help $ in the diagnosis .
-0erittin level
-;er$loplasmin
-/lphatrypsine
-Electrophoresis
225. Which will increase day care load in the ne)t decade .
--eart disease
-;/.
-:ementia
-/ccident
22'. Which one is most s$ggestive of thyroid malignancy .
-3olid cold nod$les
-;ystic cold nod$les
-Increased thyro)in level
-Increase %3- level
22?. Oo$ng &t. ;ame to $ after electric shock d$e to falling down on s$*way. C$rn was treated. E;7
is normal. 6anagement .
-3end home if E;7 is normal
-Keep monitoring the E;7 for ,5h.
-;ariad en9yme sho$ld *e checked
-E;7 m$st *e monitored for three days
22. %he most important ca$se of difference *etween 6. 8 0. mortality rate is d$e to :
-/ccident
-&$erperal
-s$icide
22#. >ower a*dominal pain with tender a*dominal mass in the cole-de-sac. :iagnosis. .
-%O/ Ft$*o-ovarian a*scessG
-Infected endometrioma
-%eratoma
(C. 3cenario is incomplete
22D. 0. 5, years hypothyroid &t. with # month ammenorrhea asking $ a*o$t osteoporosis. Oo$r
advice will *e EI;E&%:
-;yclical Est. 8 &rog.
-Ait. :.
-;a s$pplement
-;ontin$os Est. 8 &rogesterone
25!. &t. With chronic *ronchitis and &aO, of ?! mm-g. all are tr$e EI;E&%:
-;ontin$os home O, is not indicated
-/nti*iotic prophylactics is helpf$l
-Cronchodilators can *e $sed
-;orticosteroids may *e $sef$l
-(on of the a*ove
251. 3moking withdrawl which of the following is the most common symptom:
-Insomnia
-%achycardia
-/nore)ia
25,. >ife e)pectancy is increased in canada d$e to:
-:ecreased infant mortality rate
-/vaila*ility of :iagnosis and treatment
-Increased of socioeconmic stat$s
252. %he vaccine which is given to ad$lts is:
-:% pert$sis
-:%
-%
-:% and &olio
255. 6iddle aged lady came to yo$ complaining of not sleeping at night *eca$se she finished her
dia9epam ta*lets which she $sed to take *efore sleeping now her 0amily physician is away .What
will *e yo$r action:
-/ss$rance
-7ive her dia9epam
25'. &t. With frost *it@ *est treatment is:
-&$t hands in warm water 2#-5! degrees for 2! min.
-IA anti*iotics
->ocal anti*iotics and de*ridments
25?. &t. &ost 6 .I no more pain @C & D! over ?! mmhg @ p$lse is normal and reg$lar.
Whats yo$r management. .
-;lose o*servation in I ; E
-:opamin I A
-:o*$tamin I A
-:igo)in
25. &t. ;ame to yo$ with cerc$mferencial *$rn on >t. 0orearm@ complain of pain. &E: no radial
p$lsation. Cest treatment is:
-Escharectomy
-:e*ridment
-fasciatomy
25#. 6itral stenosis B$estion: lo$d 1st heart so$nd@ mid diastolic m$rm$r at the ape)@ parasternal
heave.
25D. Oo$ng female presented with vis$al impairment in one eye which improve@ in neck fle)ion an
electric like shock is felt at the *ack 8 $pper lim*. :iagnosis. .
-6ysthenia gravis
-6$ltiple sclerosis
-7C3 F7$illain-Carre syndromeG
(C. %he electric shock is called : >hermitte's sign
2'!. &t. &resent with enlarged >(. In cervical ing$inal 8 a)illary regions@ night sweat@ fever 8
weight loss. I 4ay shows widen mediastin$m. :iagnosis. .
-(-> F(on -odgkin >ymphomaG
-/6>
-I6( FInfec. 6ono. (G
2'1. %ypical case of large cervical >(. What will *e yo$r ne)t step.
-/*dominal E3
-E)cisional *iopsy
2',. Old hypertensive lady with :6. What is the *est treatment .
-C-*locker
-:i$retics
-/;EI
(C. 6;;RE 06 15. F,!!1G
2'2. 0. With *reast ;/. complains of dyspnea 8 fatig$e. &E: shows p$ls$s parado)$s. ;-I ray shows
large glo*$lar heart. :iagnosis. .
-&ericardial eff$sion
2'5. &t. with AonWille*rand disease %ype I. Which is tr$e .
-&rolonged *leeding time
-:ecreased platelet
-Increased &%.
2''. &t. on platelet transf$sion developed anaphyla)is. What is the initial management. .
-IA steroids
-3top transf$sion
-anti-histamine
2'?. &t. in I;E post 6I developed *radycardia@ p$lse at 2, L min. What is the *est management. .
-;ardioversion
-&acemaker
-IA :o*$tamine
2'. 6. , years present with one year history of heamoptysis. I 4ay shows infiltration of the l$ng.
:iagnosis. .
-4ec$rrent &$lmonary Em*olism
-Cronchiectasis
-0oreign *ody
2'#. What is the dr$g of choice of mycoplasma pne$monia .
-%etracycline
-Cactrim
-Erythromycin
2'D. %ypical case of heamolytic anemia. What is the most characteristic finding .
-4etic$locytosis
-Increased haptoglo*in
(C. findings Other than 4etic$locytosis are:
4ed$ced haptoglo*in
Increased >:-
Increased $ncon1$gated *ilir$*ine
Increased $rine *ilinogen
2?!. &t with history of alcohol intake present with icteric sclera@ Cad mo$th smell Ffetor hepatic$sG.
:iagnosis. .
--ypoglycemia
-:K/
--epatic encephalopathy
-Airal hepatitis
2?1. &t. With history of $nilat. >oss of vision in the >t. 3ide which completely recovered within '
min. What is the most likely association .
->t. Internal ;arotid /rt. 3tenosis
(C. %he :iagnosis is /ma$rosis f$ga)
2?,. Which is false .
-;efa9oline 8 Cacteroides
-7entamicin 8 E-coli
-6etronida9ol 8 Cacteroides
-Aancomycin 8 ;. difficile
2?2. Oo$ng lady with anore)ia@ generali9ed fatiga*ility@ weight loss 8 intermittent fever for , wks.
&E: m$ltiple splinter heamorragea are noted on the finger nails. %he most imp. (e)t spet is:
-Erine ; 8 3
-Clood c$lt$re
-;-Iray
(C. :iagnosis is 3$*ac$te Endocarditis
2?5. / gro$p of doctors preparing the EE. :evelop na$sea @ vomiting@ diarrhea@ few ho$rs after
having l$nch at a resta$rant. %he symptoms s$*side after ,5h. 6ost likely ca$se is :
-E-coli 0& Ffood poisoningG
-3taph. 0&
-3almonella 0&
2?'. O*ese &t. With heart *$rn@ progressive diffic$lty of swallowing 8 Epigastric pain which gets
worse more in rec$m*ency. :iagnosis. .
-/chalasia
--iat$s hernia 8 refl$) esophagitis with strict$re
-Esophageal ;/.

2??. 0. &t. With history of anore)ia@ weight loss@ vomiting@ diarrhea. C& show post$ral hypotension@
(a 12!@ K '.# @ what is the *est initial investigation .
-6eas$rment of /;%- in plasma in the morning
-/;%- stim$lation test
-/*d. ;% for adrenal
-Water deprivation test
(C. / case of /ddison's dis Fadrenocortical ins$fficiencyG.
2?. %ypical case of hepatic encephalopathy. What is the >E/3% effective in the initial management.
.
->ook for signs of 7I *leeding
-%ake history of dr$g from relatives
->ook for signs of infection
-Order lact$lose
2?#. %ypical case of migraine. What is the most likely vis$al symptoms .
-3cintillating scotoma Fthe cotton like scotomaG
2?D. /ll are se)$ally transmitted EI;E&%:
-ECA
--CA
--IA
2!. /ll are transmitted feco-oraly EI;E&%:
--/A
-ECA
-(orwalk vir$s
-&olio vir$s
(C. Kissing disease FECAG
21. %ypical case of irrita*le *owel syndrome. /ll are tr$e EI;E&%:
-(oct$rnal diarrhea
-/*dominal pain
-constipation
2,. Oo$ng 0. With s$praventric$lar tachycardia. ;omplain of chest pain@ palpitation 8 fainting.
%he *est initial management. is :
-IA verapamil
-;ardioversion
-Increase vagal tone
-C *locker
22. / n$rse in1$red *y a needle of a pt. Who is -CA FHveG 8 -IA F-veG . management..
-7ive -C. Ig. (ow 8 -C. Aaccine after , wks.
-4eass$rance
-7ive -C. Ig. 8 -C. Aaccine now
25. 7, &, with history of lower segment ;-section *eca$se of footling *reech. :$ring la*or she
present a s$dden prof$se g$sh of *right red *lood. %he fetal head is engaged. :iagnosis. .
-&lacenta &revia
-4$pt$red $ter$s
-&lacenta /*r$ptio
2'. /ll are complication of copper IE: EI;E&%:
-Eterine perforation
-Infertility
-&I:
->iver to)icity
2?. Oo$ng pregnant &t. :iagnosed to have placenta previa at ,! wks. Where the placenta covers the
internal os completely. What advise do $ give .
-/void se)$al interco$rse
-4eassesment E3 at ,# wks
-Inform the &t. %hat she needs a ;-section at delivery time.
2. Increase 03- is associated with all of the following EI;E&%:
-%$rner syndrome
-Cilat. :ermoid cyst
-&ost radiation to the pelvis
2#. Which is responsi*le for decrease O)ygen delivery to the fet$s d$ring $terine contraction .
-:ecrease art. 3$pply to the intervillo$s space
-:ecrease $terine veno$s o$tflow d$ring contraction
2D. %he most common indication for amniocentesis is :
-6aternal age is more than 2' years
-&ast history of down syndrome
-&ast history with *a*y with (%:
2#!. &t. With amenorhea@ 3he was $sing C;&. Which is helpf$l test to check the endometrial
f$nction .
-&rogesterone challenge test
2#1. Oo$ng lady in la*or@ &A shows cervical dilatation of 2 cm. partially effaced cervi)@ $terine
contraction every '-1! min. 8 lasting ,!-2! sec. What is tr$e .
-%his is an active phase
-%his is a latent phase
-/ctive phase arrest
2#,. &regnant &t. :id not feel the fetal movement for the last , days. &E: no heart so$nd co$ld *e
heard. What is the ne)t step to do .
-(on stress test
-O*stetric E3
-Ciophysical profile
2#2. &rimigravida in la*or. /ll can *e given EI;E&%:
-O)ytocin
-(arcotic analgesic
(C. Oo$ can give it only in early stage of la*or
2#5. &t. ,! wks &regnant with fi*roid present with a*dominal &ain. &E: tenderness of the lower
a*dominal Cest management. .
-/nalgesia 8 reass$rance
-6yomectomy
(C. 4ed degeneration of fi*roid.
2#'. /ll are risk factors for IE74 EI;E&%:
-/*sence of weight gain *y the mother
-0etal weight ,!!! gm at 2, wks
-6aternal maln$trition
2#?. &t. 2# wks pregnant &resented with history of clear watery vaginal discharge , ho$rs ago.
6anagement .
-E3
-3terile spec$l$m e)amination 8 take swa* for ; 8 3 Fc$lt$re 8 sensitivityG
-Ind$ce la*or
2#. Oo$ng girl after *rake $p of her relationship took 1# ta*. of lora9epam@ she only complains of
di99iness 8 drowsiness. /fter ? h. of o*servation she is well@ feel remorse 8 want to *e discharged.
6anagement .
-:ischarge her 8 follow $p as o$t pt.
-0$ther ,5 h. o*servation
-;all her *oyfriend
2##. Oo$ng girl fo$nd in coma in the street. &E: constricted p$pils Fpin pointG 8 m$ltiple marks of
IA in1ection. 6ost likely ca$se is :
--eroin
-;ocaine
-:ia9epam
2#D. ;ase of dr$g over dose with fever@ tachycardia@ dilated p$pils 8 C&: ,,!L11!. 6ost likely ca$se
is:
-;ocaine
->3:
--eroin
-Car*it$rates
2D!. &sychic str$ct$re that relate desire to e)ternal environment is :
-I:
-Ego
-3$perego
-3elf
-&ersonality
(C. %o *e checked ...
2D1. /ll are tr$e regarding O;:. EI;E&%:
-Ego dystonic
-Ego syntonic
(C. :ystonic < O; :isorder.
3yntonic < O; &ersonality.
2D,. Which is tr$e regarding /l9heimer's disease .
-/pra)ia@ agnosia@ dysphasia
-3$dden onset
-6ore common in male
(C. /l9heimer's 0. are more than 6. 8 its not with s$dden onset
2D2. Which of the following ca$se dr$g ind$ced dementia .
-:ia9epam
-Imipramine
-%ria9olam
2D5. &sychotherapy s$perior to medication in
-3chi9ophrenia
-/lcoholism
-:r$g ind$ced deliri$m
-dysthimia
2D'. Which is tr$e regarding s$icide in adolescent .
-6ore common in s$mmer
-/ttempt to s$icide is more common in 0. %han in 6.
-6ore common in 0.
2D?. ;hild D years old st$ding in kindergarten@ is $na*le to read@ write or even to color a pict$re. -e
*ecomes happy when he answers simple B$estions. What is yo$r diagnosis .
-/$tism
-6ental retardation
-3pecific learning disa*ility
2D. What is the *est management. of school pho*ia .
-0orce him to go to school
-Keep him at home $ntil he miss his friends in school
-7ive him an)iolytics
2D#. Which is regarded as an aro$sal symptom of &%3::
-4estricted affect
-0eeling of detachment
--ypervegilance
2DD. &t. With m$ltiple tra$ma came to hospital shocked *$t he is agitated 8 violent. 6anagement
4egarding treatment .
-Wait $ntil he *ecomes more shocked
-3edate him 8 treat
-%reat him immediately even if yo$ may physically restrain him

5!!. 6. 22 years present with s$dden onset of mood 8 memory changes@ m$te.on admission he
developed conv$lsions. :iagnosis. .
-/I:3 :ementia comple)
5!1. %he most imp. ;a$se of increased complications of measles in developed co$ntries is :
-InadeB$ate imm$ni9ation
-InadeB$ate n$trition
5!,. 3t$dy has *een done *etween risk factor 8 a disease@ Ods ratio was ,.2. what is tr$e .
-%here is significant association *etween risk factor 8 disease.
-%he disease is ,.2 times more in those having risk factor
5!2. Which is the most practical meas$re to red$ce occ$pational deafness .
-%he $sage of daily prophylactic ear pl$gs d$ring d$ty
-6eas$ring the noise freB$ency
-Esage of ear protection
5!5. /ll are tr$e regarding incidence EI;E&%:
-%ake only the new case in consideration
-Estimate the risk of acB$iring the disease In comm$nity
-Esef$l meas$re of disease &ro*lem in comm$nity
-Esef$l for *oth ac$te 8 chronic disease
5!'. Which of the following is tr$e regarding non Ioni9ing radiation .
-;an ca$se ;hronic *ronchitis
-;an ca$se >oss of vision
-;an ca$se cataract
5!?. Which is not tr$e regarding O2 .
-Increased aro$nd photocopy machine
-;an *e harmf$l at gro$nd level
-Its mainly d$e to incomplete com*$stion of fossil f$els
5!. In place*o treatment . /ll are tr$e EI;E&%:
-4esponse to place*o indicate that the ca$se is psychogenic
-4epeated $se decreases its efficacy
-1L2 of those with organic ca$ses respond to the place*o
5!#. /ll are tr$e regarding the role of physician in tra$ma pt. EI;E&%:
-/ssess the pt. ;ondition clinically
-;an determine the d$ration of treatment
-Estimate the compensation of the pt.
5!D. Information a*o$t mortality in ;anada can *e o*tained from
-;anada statistics
-&$*lic health center
-O$t pt. clinic
51!. ;hild 5 months old age. -e m$st get his second vaccination. -is mother is worried a*o$t the
risk of fever. What is yo$r advice .
-7ive anti*iotic if temp. 6ore than 2D ; .
-/3/ if temp. rises
-7ive acetaminophen now 8 Bid later .
-;all the doctor if temp. 4ises more than 2D ;
511. Ca*y 5 months of age. 6icrocytic hypochromic anemia. What is the
;a$se .
-Creast feed only
--is mother was taking erythromycin d$ring pregnant
-&remat$rity
51,. ;hild with microcytic hypochromic anemia. -is mother *lood film shows *asophilic stippling 8
microcytic hypochromic anemia. 3imilar finding were fo$nd in the father. (e)t step .
-;C;
-3er$m ferritin
-&acked plasma
--C electrophoresis
512. /n /frican 5 years old child@ with dark $rine@ treated for a respiratory tract infection *y
3eptrim@ present with 1a$ndice 8 pallor. What is the diagnosis. .
-7?&:
-;ycle cell anemia
-%halassemia
515. / ;hlid with />>. Will present with all of the following EI;E&%:
-/nemia
-Infection
-%hrom*ocytopenia
--eamoglo*in$ria
51'. (ew*orn 5 days old@ with poor feeding@ mottling skin@ temp. 2?.'@ after sta*ili9ing the *a*y@
what is the ne)t step .
-/C7 Fart. *lood gasG
->&
-Electrolytes
51?. ;hild with sever vomiting 8 diarrhea @ *ecame lethargic with s$nken eyes@ mottling skin@ low
C&@ Increase p$lse@ (a 1'! mmol @what is yo$r management. .
-&lasma 1! ccLkg within 1st h.
-'" de)trose ,! ccLkg. Within 1st h.
-!.D normal saline ,! ccLkg. Within 1st h.
51. ;hild with moderate to severe dehydration. Which of the following investigation helps in yo$r
management .
-;C;
-Clood $rea
-Electrolytes
51#. ;hild with severe dehydration (a 1! mmol. What is the complication e)pected d$ring yo$r
management. .
-3hock
-:eath
-;onv$lsions
(C: conv$lsions may happen d$e the rapid correction of hypernitremia. In hyponitremia cere*ral
edema may *e developed d$e to rapid correction
51D. /ll can ca$se a*dominal 6ass in neonate EI;E&%:
-Wilm's t$mor
--ydronephrosis
-(e$ro*lastoma
-6eckl's divertic$l$m
5,!. What is tr$e regarding congenitla pyloric stenosis .
-;ommonly present at 2 months
-/ssociated with meta*olic acidosis d$e to vomiting
-Aisi*le peristalsis is seen in a*domen
5,1. ;hild with nephrotic syndrome presented with fever@ a*d. &ain 8 ascitis. What is yo$r ne)t step
.
/*d. Iray
E3
&arasynthesis
5,,. / child presented with fever 8 small white lesion on the m$co$s mem*. of the mo$th followed
*y generali9ed mac$lar pap$lar rash. What is the manag. .
7ive /3/ to decrease fever
7ive gamma Ig.
(otify the p$*lic health $nite
Isolation of the family mem*er
(C: diag. Is 6easles
5,2. ;hild 1' yrs present with C&. 1''LD! mm-g. What is yo$r manag. .
4estrict salt in diet
:i$retics
4epeat meas$rement of C& at different times
-aloperidol
5,5. -ow can yo$ diag. / , yrs old child with an -IA FHveG mother .
Erine analysis
;C;
;:5L;:# ratio
E>I3/
5,'. ;hild 1, yrs present with constipation @ decreased school performance 8 s$ppressed growth for
the last 1, months. :iag. .
3chool pho*ia
-yperthyroidism
-ypothyroidism
5,?. (ew*orn 1! days developed $ncon1$gated hyper*ilir$*inemia@ he was *orn at term. 8 he is
otherwise well. :iag. .
7alactosemia
Kernicter$s
Creast milk 1a$ndice
5,. ;hild present with sore throat@ fever 8 diagnosed as Infectio$s monon$cleosis. -e was treated
conservatively *$t he ret$rned to $ with a severe pain in the throat. 6anag. .
7ive ampicciline for 1! days
%hroat swa* for ;83
Iray
5,#. ;hild 1# months old present with generali9ed conv$lsions for ,! min$tes. -e has history of E%I@
no history of similar attack *efore. /fter he was given O, what will *e the appropriate step in the
mana. .
4ectal dia9epam
4ectal paraldehyde
&heno*ar*ital I6.
5,D. &reterm *a*y with feat$re s$ggest Iron def. /nemia. When will $ give Iron s$pplement .
2 months of age
' months of age
5 wks of age
52!. ;hild D yrs old with en$resis. Its $s$ally associated with:
a part of generali9ed an)iety disorder.
E%I
521. -ow to differentiate *etween divertic$losis 8 pro)imal ;/ in narrowed segment of colon .
-;olonoscopy 8 *iopsy
-3igmoidoscopy
-Cari$m enema
52,. &t. ;omplains of constipation@ a*dominal pain 8 freB$ent diarrhea mainly in the morning 8
with (O other pro*lems. :iag. .
-IC3 Firrita*le *owel syndromeG
-:ivertic$losis
522. &t. With ple$ral eff$sion on the 4t. 3ide with no apparent lesion. 6anag. .
-%horacocentesis
-%horacotomy 8 *iopsy
-;%
525. :r$g ca$sing mania EI;E&%:
-3teroid
-;imetidine
-Imipramine
52'. What is the most helpf$l sign in diagnosing ing$inal hernia in children .
-%hickening of the cord
-Wide e)ternal ring
-C$lging at the groin
52?. &t. ;omplain of headache@ vomiting following 4t. -emianopsia. :iag. .
-6igraine
-/ma$rosis f$ga)
-4t. 4etinal art. stenosis
52. >iver cirrhosis pt. ;omes with massive heamatemesis. What is the *est ne)t step .
-Ait. K IA.
-0actor II @ AII @ II @ I
-7ive packed FwholeG *lood transf$sion
52#. Which is not tr$e in a pt. With mycoplasma pne$monia .
-0ever @ persistent hacking co$gh
--eadache@ diarrhea 8 non e)$dative pharyngitis
-/ssociated with erythema m$ltiformis
-/ssociated with *$llo$s myringitis
-;an ca$se rapid progressive pne$monia with cyanosis
(C. Its the walking pne$monia in yo$ng man ...
52D. (ew*orn pale@ -4 < #!Lmin. 4espiration is Irreg$lar 8 slow@ fla**y@ la9y@ 8 he is weak for
irrita*ility. What is the /&7/4 score .
-,
-2
-5
-'
-1
55!. &t. With syphilis 8 FHveG A:4> 1:,! *efore. -e receives penicillin for , months 8 the titer is
1:? now. Which of the following is tr$e .
-7ood response to treatment
-:ecreasing in titer sho$ld *e more faster
-&t. Is imm$nos$pressed
-Cacteria develop resistance to penicillin
551. Which of the following *enefits from vasodilatation .
-;or p$monale
-3$*aortic stenosis
-6itral ins$fficiency
-6itral stenosis
-/3:
55,. /*sol$te contraindication of C;&. Is :
--istory of &$lmonary em*olism
-6igraine
--%.
552. Which is tr$e regarding the diag. of *rain death for organ transplantation .
-/*sence of all spinal refle)
-EE7 completely for 5# h.
-;o$ld *e diagnosed clinically
555. Ait.: deficiency associated with all EI;E&%:
--ypocalcemia
--yperphosphatemia
--igh />& Falkaline phosphataseG
55'. Aietnamese &t. -as 1' mm. FHveG 6anto$) test F%$*erc$lin testG 8 F-veG cytology. 6anag. .
-%reat the pt. witho$t regard to sp$t$m cytology
-4epeat 6anto$) test after 2 years
55?. %reatment of pt. With dystonia is :
-%;/
-6/OI
-Cen9tropine
--aldol
55. (ew*orn with small head@ small palpe*ral fiss$re@ small philtri$m 8 small eyes 8 flattened
meat facial area. :iag. .
-0etal alcohol synd.
-:own synd.
-Edward synd.
-%$rner synd.
55#. ,! wks &regnant with family history of :own synd. Which of the following sho$ld *e done to
r$le o$t :own synd. .
-/mniocentesis
-;horionic villo$s sampling
-/*d. E3
55D. Which of the following is the treatment of choice of 4he$matic 0ever .
-/spirin orally
-Cen9athine penicillin 7 F1., million IEG I6.
-&enicillin IA.
-;eftria)one IA.
5'!. 3ickle cell anemia with fever 8 severe a*d. &ain. 6anag. .
-IA fl$id@ *lood c$lt$re@ anti*iotics
-IA fl$id@ narcotic@ *lood c$lt$re
-Clood transf$sion
5'1. &t. With *ilat. >ower lim* pain e)aggerated *y night@ noct$ria 8 no history of $rine
incontinence. Investigation reveals gl$cos$ria. :iag. .
-6assive cord compression
-:ia*etic peripheral ne$ropathy
--erniation of spinal disk
5',. /ll are tr$e regarding folate deficiency EI;E&%:
-6;A < D?
-(ormal ser$m folate
-%hrom*ocytopenia
(C: 6;A has to *e more than 1!!
We check the 4C; folate 8 not the ser$m folate.
5'2. %he company need to change the dipstick for detection of protein$ria *eca$se it was detecting it
in low threshold F(o. of false positive was highG. (ow it increases the threshold of the screening test
to detect protein$ria. %his will affect the test in terms of :
-Increased sensitivity 8 specificity
-Increased (o. of 0&.
-Increased (o. of 0(.
(C. Increasing the threshold will res$lt in decreased sensitivity 8 increased specificity.
5'5. Cest treatment for :6. type I is:
-3hort acting ins$lin H (&-
-3hort acting ins$lin H long acting ins$lin
-Etra acting ins$lin
-(&- H long acting ins$lin
(C. 6;;R E?. F,!!1G
5'?. &t. 4et$rned from vacation he is hypertensive on thia9ide. -e now complains of pain@ swelling
of 4t. Knee 1oint 8 fever. Cest manag. Is .
-Knee aspiration
-3er$m $ric acid level
(C: thia9ide may ca$se increase level of Eric acid which can ca$se the swelling of the 1oint
5'. 4/. &t. on gold salt H hydrochloroB$ine develop protein$ria 8 ascitis. What is the ca$se of his
condition .
-:$e to his chronic dis.
-:$e to 7old salt
-:$e to hydrochloroB$ine
5'#. Which is tr$e regarding /co$stic ne$roma .
-:ecrease *one cond$ction in the same side
-Increase *one cond$ction in the same side
-Increase *one cond$ction in the same side
5'D. In 4t. 3ero$s otitis media@ We**er will give :
-:ecrease *one cond$ction in the 4t. Ear
-:ecrease *one cond$ction in the >t. Ear
-Increase *one cond$ction in the 4t. Ear
5?!. %he most predisposing factor to -%. is :
-/lcohol
--ereditary
-3moking
-O*esity
5?1. Which is the least ca$se of -IA infection :
--omose)$ality
-&rostit$tion
-:r$g a*$se F1DDDG
-Clood transf$sion F1D##G
5?,. -ypotonia@ hypoglycemia@ hepatosplenomegaly 8 poor feeding infant. :iag. .
-3epticemia
-7lycogen storage dis.
5?2. &regnant 15 wks with hydatid mole. Cest treatment is .
-:ilatation s$ction c$rettage
--ysterectomy
--ysterotomy
-6ethotre)ate
5?5. &t. ;ame with m$ltiple tra$ma. Which will $ treat 1st .
-%ension pneomothora)
-;ardiac tamponade
-/ortic r$pt$re
-3k$ll fract$re with visi*le *rain
5?'. &t. With m$ltiple fract$re in I;E. -e developed dyspnea@ increased ;O,@ decreased O,. Iray
shows *ilat. Infiltration F/4:3G. 6anag. .
-Int$*ation 8 ventillation
-O, mask
--;O2 IA.
5??. In se)$ally a*$sed ;hild. Which will *e the most s$ggestive sign:
-A$lva *r$ise
->aceration on the la*ia
-4$pt$red hymen
->a*ial aggl$tination
5?. &t. With *reast ;/ $nderwent operative s$rgery 8 chemoradiotherapy@ she developed
pne$monia followed *y death. What is the $nderlying ca$se of death.
-;ytoto)ic chemotherapy
-Creast ;/
-&ne$monia
5?#. What is the most dangero$s complication of ma)illary sin$sitis .
-&erior*ital cell$litis
-;ervical adenitis
5?D. What is the treatment of %o$rettes syndrome .
--aldol
-0le$)etine
->ithi$m
-;ognitive *ehavioral therapy
5!. Which is tr$e regarding a childhood 3chi9ophrenia .
-:iagnostic criteria like ad$lts
--all$cinations 8 deliri$m are rare
51. :ysmenorrha in an 1# yrs old is mostly associated with:
-(ormal pelvic e)am
5,. 4etroverted $ter$s is mostly associated with
-Endometriosis
->ow *ack pain
-Infertility
52. /ll of the the following can *e in the manag. hypercholestorelemia EI;E&%:
-;onservative for 5 to ? months
-3imvastatin
-3erevastatin
->ovastatin
->ipos$ction
(C. Withdrawn from the market in ;anada in /$g$st ,!!1 d$e to increase risk of death with this
3erevastatin.
55. Which of the following will *e a good treatment for ovarian t$mor according to its adeB$acy .
-3$rgery
-4adiotherapy
-;hemotherapy
5'. &ost-pelvic radiation pt with diarrhea. What is the most common ca$se .
-proctitis
5?. -ydronephrosis associated with all EI;E&%:
-&ost-$retral valve
-O*str$ctive $ropathy
-C&-
-(ephrotic syndrome
5. Which is associated with dys$ria *$t no p$s cells in $rine .
-E%I
-A$lvovaginitis
5#. Which is not associated with occ$pational asthma .
-&olyvinyl chloride
-Isocyanate
-/nhydride
-/nti*iotics
-resin
(C: polyvinyl chloride ca$ses liver 8 *rain ;/.
5D. 3e)$al a*$se is associated with all of the following EI;E&%:
-:evelopment of secondary se)$al characteristics
-6ost of assailants are known to the victim
-6ore common in 0. %han in 6.
-3pecific or generali9ed fears@ depression 8 night mares
5#!. Which is false regarding anencephaly in pregnancy:
-3ampling of $m*ilical art. Will not reveal the diag.
-5 mg. 0olic acid is prophylactic
--igher dose given for a pregnant who has history of anencephaly
-/mniocenthesis $sed in diag.
-/ssociated with high alpha fetal protein in mothers ser$m
(C: the correct dose sho$ld *e F!.5 - 1 mgG in all pregnant women 8 we increase the dose to 5 mg if
the pregnant has a history of (%:.
5#1. Which is contraindicated in the last trimester .
-3$lfonamide
-/mpicillin
-:ia9epam
--eparin
(C: increased risk of kernicter$s
5#,. What will *e the effect of :ia9epam taken *y a *reast feeding mother on -her child .
--ypotonia
--yper ventilation
-Intra ventric$lar hemorrhage
5#2. Which of the following is contraindicated on pert$sis vaccine .
-Egg allergy
-Encephalitis after the previo$s dose
-Epper respiratory tract infection with slight fever
5#5. ;hild with *loody :iarrhea with dehydration can *e managed *y all EI;E&%:
->operamid
-6etronida9ol
-4ehydration
-3tool c$lt$re
5#'. What is the complication ca$sed *y forceps delivery .
-;ephalohematoma
-;ap$t s$ccidan$m
5#?. ;omplications ca$sed *y Aacc$m delivery is :
-;ephalohematoma
-;ap$t s$cidan$m
-/ll of the a*ove
(one of the a*ove
5#. Which is not tr$e in IE74 FIntra$terine 7rowth 4etardationG :
-/*sence of weight gain
-6aternal short stat$re
->ow f$ndal height
5##. / st$dy reveal that 1! yrs e)perienced driver contri*$ted to ?!" of accident. ' yrs e)perienced
driver contri*$te to 2!". 8 those with , yrs contri*$te to 1!". %he concl$sion was that the more
e)perienced driver are more careless in driving. %his concl$sion is not right *eca$se of :
-It's not meas$ring the incidence of accident
-It's not standardi9ed to the age
-Ceca$se there is no control gro$p
5#D. 3t$dy a*o$t the *enefit of e)ercise in decreasing the risk of heart dis. /pplied to a gro$p of
vol$ntaries in the factory with a control gro$p of the rest of the factory. 0ollow $p was reg$lar. %he
check $p shows lower rate of I-:. Ischemic -eart :is. In e)ercising gro$p Fvol$ntariesG. %he
concl$sion that e)ercise protect against I-:. may *e re1ected *eca$se of:
-3election *ias
-Only 6. Were incl$ded
-Only one company was involved
-3ome I-: may *e silent
5D!. :ifferentiation *etween schi9ophrenia 8 mania:
-%ho$ght *roadcasting
-0light of ideas
5D1. Which is false a*o$t :own syndrome .
-4ocker Cottom 0oot
-Cr$shfield spot
-Epward standing palpe*rale fiss$re
-Crachiocephaly
5D,. In 1' wks. (ormal pregnancy $ will find all EI;E&%:
-Increased C&
-0$ndal height midway *etween symphysis p$*is and $m*ilic$s
-Increased heart rate
5D2. Opiate a*$se is commonly associated With:
-6iosis
-E$phoria
-&erforation of nasal sept$m
5D5. Old pt. /*$sed *y his wife complain to $@ yo$ will do all the following EI;E&%:
-3end the pt. %o day care
-0reB$ent visit to ho$se
-4eass$rance
5D'. &t. :ied after s$ffering from %C@ what will yo$ write in death certificate as ca$se of death .
-;ardiac arrest
-%C
-0ever
->oss of weight
5D?. /:-: associated with all the following EI;E&%:
-3tereotyped movement
-3hort attention span
--yperactivity
-Imp$lsivity
-Interference with family 8 social f$nctioning
5D. Which of the following is contraindicated with tyramine rich food FcheeseG .
-Imipramine
-&henel9ine
-0le$)itine
5D#. Which increases the risk of 6I. .
--istory of father with 6I
--istory of smoking
--istory of high animal fat diet
5DD. &t. With chronic pain@ can *e treated with all EI;E&%:
-Ciofeed *ack
-Operant conditioning
-:esensiti9ation
'!!. Which of the following is most compati*le with interstitial l$ng dis. .
-3evere hypo)ia on e)ercise Fdecreased &aO,G
-Emphysema
-Increase ;O, 8 O,
'!1. What will *e yo$r ne)t step on a pt. with normal *reast at &E. 8 -/*normal mammogram .
-Ciopsy g$ided *y wires
-E)isional *iopsy
-4epeat after one year
'!,. /ll are part of the normal vaginal flora EI;E&%:
-&eptostreptococ$s
->acto*acill$s
-3taphylococ$s
-;6A
'!2. Which is the most likely to *e associated with >e$kemia .
-/romatic hydrcar*ones
-Ioni9ed radiation
-Cen9oalphaperines
'!5. 3$rveillance in medical worker for radiation ha9ard incl$de all EI;E&%:
--eamatological e)amination
-/nn$al &E.
->a*elled panding
-%otal *ody radiation co$nt
'!'. /ll are tr$e regarding aseptic meningitis EI;E&%:
-Increased chloride
-(ormal *lood s$gar
-Increased lymphocyte
-3light protein increase
'!?. :ysplastic nevi associated more with:
-3$*$ng$al
-Elevated
--ypertrichosis
'!. :rainage of lymph in lower v$lva:
-Ing$inal >(.
-Internal iliac >(.
-&ara/ortic >(.
'!#. C;& red$ces risk of which of the following .
-Creast ;/
-Endometrial ;/
'!D. %he most common complication of low dose of C;& is:
-Crake thro$gh *leeding
'1!. &t stop her C;& 8 ask yo$r advice for time of conceiving. What will $ tell her .
-Cody tepm. ;hart
-&regnancy after ? months
-Cleeding will occ$r within ,# days
-%ry to conceive after she got the menstr$ation
'11. Which is the *est so$rce of information for pt. With poisoning *y a to)ic s$*stance .
-&$*lic health $nit
-Environmental health protection $nit
-%o)icology $nit
'1,. Which is tr$e regarding $rticaria .
-In #!" of cases the ca$se is not apparent 8 It'll resolve spontaneo$sly
'12. 4egarding smoking cessation. /ll are tr$e EI;E&%:
-:ecreased l$ng ;/ occ$rrence as other nonsmoking pop$lation *y ,-2 years of cessation
-Improvement of cla$dication
'15. 7$n shot to the lower a*domen@ pt. Is in shock with C& !L2! what is yo$r immediate
management. .
->aparotomy
-&eritoneal lavage
'1'. In c$shing syndrome yo$ find all EI;E&%:
->ow C&.
-O*esity
-3triation of skin
-%r$nk o*esity
'1?. Old pt. With history of head tra$ma@ he has changes in *ehavior with freB$ent vomit 8
headache. E do /ll EI;E&%:
--ead ;%
-;atheter
-IA. 0l$id
->&
'1. F;ase of Invalid consent eg.G %aking consent from a premedicated pt. On the way to theater for
some proced$re.
'1#. %hia9ide is *est ro *e given to old hypertensive pt. With :
-Aasovagal attack
-:6.
-7o$t
'1D. &t. With s$prachondilar fract$re with splint 8 he is in complete fle)ion. -e develops ischemia.
What will *e the initial management. .
-4emove splint 8 fle) the arm
-Keep splint 8 e)tend the arm
-4emove the splint 8 e)tend the arm
-/ngiogram of *rachial art.
',!. &t. Who ate CCR. :eveloped diarrhea. Clood co$nt shows eosinophelia. %he *est investigation
to *e done to reach the diagnosis .
-6$scle *iopsy
-3tool c$lt$re
-3tool microe)amination
',1. &regnant came with *reech presentation at 2D wks. /ll other parameters are normal.
6anagement .
-;ontin$e /(; Fantenatal careG
-;-section
-(ormal vaginally delivery
',,. Which is the most indicative of fatal distress:
-Case line heart rate at 11' *eatLmin
-Occasional late deceleration
-Early deceleration
',2. &t. On 1!!" O, 8 ventilator.
&aO, < D2"
&a;O,<'?"
p-<.2 -;O2<21
What will *e yo$r management. .
-:ecrease O, sat$ration
-Increase respiratory rate
-Increase tidal vol$me
',5. >aryngeal polyp can *e ca$sed *y all of the following EI;E&%:
-&resent medication
-3moking
-/llergy
','. / secretary complains of painf$l lesion .' cm a*ove the an$s in the anal cleft. :iagnosis. .
-Infected pilonidal sin$s cyst
(C. 6;;R ,!!1 F73,?G
',?. &t. With sigmoid volv$l$s. 6anagement .
-3igmoidectomy
-:ecompression *y sigmoidoscope
--ydrostatic *ari$m enema
',. &t. &resent with dyspnea 8 fever 2#.' @ 2?h. &ost-op. :iagnosis. .
-:A%
-/telectasis
-&$lmonary Em*olism
',#. What is tr$e regarding mastitis .
-Es$ally treated *y I8:
-%emp. (ever more than 2D
-/ssociated with s$pressed lactation
-Es$ally start ,-5 wks. &ostpart$m
',D. Which of the thyroid ;/. -as the *est prognosis .
-0olic$llar
-&apillary
-/naplastic
-6ed$llary
-Em*rionic
'2!. C$rned &t. &resented $nconscio$s to E4. &E: smoke in the mo$th@ *lack tong$e@ intact pharyn)
8 he was (O% cyanotic. What do $ e)pect to find .
->ow &aO,
-;ar*o)yheamoglo*ine more than 2'"
-&a;O, < '? mm-g.
(C. %o *e checked ...
'21. &ost-op pt. &resent with hypotension 8 central veno$s press$re 1? cm -,O. :iagnosis. .
--ypovolemic shock
-;ardiogenic shock
'2,. /ll are tr$e regarding thoracic dissecting aortic ane$rysm EI;E&%:
-3evere chest pain
-3ystolic m$rm$r@ when present@ is of great significant
-Involvement of aortic arch may lead to stroke
-/symmetrical peripheral p$lses
'22. 6ost common indication of cricothyroidotomy is:
-6$ltiple severe facial in1$ry
-0ract$re ;'-;? with dislocation
'25. /nal skin tags in a child is commonly associated with:
-;rohn's disease
-;hronic anal fiss$re
--emorrhoids
'2'. >ady ,! years presented with 2 cm mo*ile *reast mass@ it was increasing in si9e for the last 5
months. :iagnosis. .
-0at necrosis
-0i*roadenoma
-0irocystic disease of the *reast
'2?. 0. pt. &resented with history of a*dominal &ain @ fever @ vomiting @ the pain shifted to the 4t.
>ower B$adrant. %he pt. Improved after a conservative treatment. 1! days after he present with 4.
a*dominal 6ass. :iagnosis. .
-/ppendicitis
-/pendic$lar a*scess
-/c$te s$lpingitis
'2. Which of the following is tr$e regarding e)traintestinal manifestation of an IC: of $nknown
origin .
-%otal colectomy res$lt in relief of e)traintestinal signs 8 symptoms
-E)traintestinal symptoms may precede gastrointestinal symptoms
-/rthritis $s$ally involve small 1oints
-/rthritis are more
cTPYMUVWWWWXSTQYMUVWWWWXSTPYMUVWWWWXSTQYMUVWWWWXSTPYMUVWWWWXSTQYMUVWWWWXSTPYMUVWW
WWXSTQYMUVWWWWXSTPYMUVWWWWXSTQYMUVWWWWXSTPYMUVWWWWXSTQYMUVWWWWXSTPY
-3top antihistamine ,-5 wks *efore
-3top C;& 5-? wks *efore
-;ontin$e *oth dr$gs
(C: C;& will increase the risk of hypercoag$la*le state
'51. What is 6allet finger .
-0i)ed fle)ion of :I&. With loss of active e)tension ca$sed *y r$pt$re of the e)tensor tendon.
(C: 6;;R ,!!1 F&>DG
'5,. /ll the following occ$rs in intestinal o*str$ction EI;E&%:
-:istention
-Increased *owel so$nds
-7$arding
'52. &ost-op lady complain of red painf$l swelling@ calf is free of pain. :iagnosis. .
-:A%
-3$perficial throm*ofle*itis
'55. %he most consistent finding in 0etal /lcohol 3ynd. is:
-6icrocephaly
'5'. Oo$ng *oy with a*dominal 6ass not moving with respiration. /ll can *e diagnosed EI;E&%:
-Wilm's t$mor
-(e$ro*lastoma
-:o$*le 1e1$n$m
-6eckel's divertic$l$m
'5?. ;holostrom protects infection *y:
-Ig6
-Ig/
-Ig7
'5. ;hild have ingested ,! ta*lets of Iron. /ll can appear EI;E&%:
-(ecroti9ing Enterocolitis
-3hock
-6eta*olic acidosis
-;oma 8 death
-;ere*ellar involvement is an early symptom
'5#. &t. &resent increased />& Falkaline phosphataseG 8 normal ;aHH . :iagnosis. .
-&agets dis of the *one
-3c$rvy
--yperphosphatemia
'5D. /ll are irritant to m$c$s mem*. EI;E&%:
-3O,
-(O,
-;O
''!. In coarctation of the aorta we find all EI;E&%:
-:elayed radial femoral p$lse
-Increase $pper lim* C&.
-;ontin$es m$m$r in the chest
''1. What is the most imp. risk factor for stroke .
-3moking
-3ystolic -%.
'',. What pop$lation gro$p is e)pected to increase in ;anada ne)t years .
(eonates
-1!-1' years.
-,!-5! years.
-5!-?' years.
-/*ove ?' years.
''2. %he prevalence of a recessive gene is 1:5!!!!. What is the prevalence of the disease In
comm$nity .
-1:,!!
-1:5!!
-1:5!!!
(C: sB$are root of the 1:5!!!! < 1:,!!
''5. What is the policy of ;anada to decrease the health e)pendit$re .
-/dvise the health professional to $se the health reso$rces
'''. &t. &resented with cardiac arrest. Which descri*es the *est action .
-IA lidocaine ro$tinely given
-IA /drenaline every 1! min.
-;hest compression will keep circ$lation for cere*ral flow
-/tropine every , min.
''?. 0. &t. ;omlain of paralysis of the 4t. 3ide of *ody after divorce. What is yo$r diagnosis .
-;onversion disorder
-6alingering
-0actitio$s disorder
''. ($rse with repeated attack of hypoglycemia. Investigation reveal increased ser$m ins$lin *$t
low ser$m ;-peptide. 6anagement .
-7- test
-7%% Fgl$cose tolerance testG
-&sychiatric assessment
''#. Er*'s palsy associated with
3ho$lder distocia
''D. %he mane$ver of 0le)ion of the thighs against the hips is helpf$l in the delivery of:
-3ho$lder dystocia
'?!. %he most common ca$se of late postpart$m hemorrhage is
-Eterine atony
-Evac$ation of vaginal hematoma
-3$*invol$ted $ter$s
-;ervical laceration
'?1. >ady after a prolonged la*or she delivered a 5 kg. (eonate. 3he is not a*le to $rinate.
:iagnosis. .
-Erethral tra$ma
-6aternal dehydration
-Eterine atony
-Cladder atony
'?,. &ostpart$m lady present with fail$re to lactate@ generali9ed weakness@ yo$ do all the following
EI;E&%:
-%3-
-%5
-03- 8 >-
-prolactine
'?2. Individ$al " of infertility@ what is the highest .
-Eterine a*normality
-6ale factor
-Ovarian ca$se
-;ervical ca$se
'?5. &rimigravida in la*or@ fet$s at ! station@ cephalic presentation@ occip$to-transverse@ cervi) 5 cm
dilated for ? h. Efficient $terine contractions every '-1! min. lasting 1! sec. What is yo$r diagnosis .
-O*sr$cted la*or
-(ormal latent phase
'?'. &ropranolol is $sed in all EI;E&%:
-Cipolar disorder
-/ggressive violent disorder
-3ocial pho*ia
->ethi$m tremor
-/cathesia
'??. %he nat$ral growth rate <
-Cirth rate - immigration rate
-Cirth rate - death rate H immigration
-Cirth rate H immigration rate
'?. While yo$ are waiting for ;$lt$re 8 sensitivity test. What will yo$ treat shigellosis with .
-/mpicillin
-%6&L36I
-Erythromycin
'?#. ;hild with C& 1'!LD! mm-g. Oo$ s$spect coarctation of /orta. What is the most common
finding .
-Increase C&. In 4t. Epper lim* than in the >t. Epper lim*
-Increase C&. In lower lim* more than Epper lim*
-Increase C&. In the Epper lim* more than the lower
'?D. &t. With :6. &resented with history of diarrhea. What is the most likely ca$se .
-/$tonomic diarrhea
-Infectio$s diarrhea
-Inflammatory diarrhea
'!. Oo$ng single 6. With history of dementia@ ata)ia 8 cognitive disorder. :iagnosis. .
-:epression
-/I:3 related dementia
-/l9heimer's disease
'1. 0. ;ame with history of fever 2D @ a**. pain. &E: fe*rile@ *ilateral lower a*dominal pain with
>t. side 6ass tender. One week ago pt. was diagnosed with clamedia *y c$lt$re 8 sensitivity. Whats
yo$r approach to treat her .
-;eftria)one
-Oral :o)ycycline
-O$t patien oral :o)ycycline 8 I6. ;eftria)one
-Inpatient oral :o)ycycline 8 IA. ;eftria)one
',. &t. With repeated a*ortion. /ll are possi*le ca$ses EI;E&%:
-:ecreased l$theal phase
-3eptate $ter$s
-Incompetent cervi)
-4$pt$re heamorrhagic corp$s l$thi$m
'2. &t. came with an ac$te episode >t. 4enal colic@ IA& shows >t. ;alc$li at the $reter at the level of
>.5 . %here is small dye leak *eyond the calc$li@ >t. Kidney enlargement. 6anagement .
--ydration
-O*servation
-4emove the stone *y retrograde cystoscopy
->ithotripsy
'5. Oo$ng man came with history of a$tomo*ile accident@ m$ltiple anterior chest fract$re and
hoarseness @ chest )-ray shows widened mediastin$m@ whats yo$r immediate management.
-Int$*ation
-3$pport the anterior chest fract$re
-&ericardial synthesis
-/ortic angiogram
''. 3chi9ophrenic &t. on phenothia9ine came with history of gremacing@ p$rposeless movements of
the lim*s@ :iagnosis. .
-%ar dive :yscanasia
-/cathesia
-(e$rolyptic 6alignant 3yndrome
'?. / known &t. With *ipolar disorder *ro$ght to $ with history of agitation @ paranoid@ del$sion@
and grandiosity . Whats yo$r action .
-Cen9odia9epen
->ithi$m and /ntisycotic
-/ntisycotic
-/ntidepressant
'. &t. ;ame to $ after he divorced his wife@ he has history of depressed mood@ which of the
following factors prompts yo$ to start antidepressant .
--istory of divorcing *efore
--istory of mother and sister treated with antidepressant
--istory of alcohol
'#. What is the commonest complication of :%. .
--eart fail$re
4enal fail$re
4espiratory fail$re
'D. /ll are tr$e regarding -antington disease EI;E&%:
Onset at mid ?!'s
/$tosomal dominant
&rogressive disease
/ssociated with depression
/ssociation with dementia
(C. %he normal age of onset is *etween 5!'s 8 ?!'s
'#!. In thyroto)icosis all are tr$e EI;E&%:
/menorrhoea
Weight loss
;hange of appetite
'#1. %he *est time to amniosenthesis is:
1? wks
1, wks
,! wks
2! wks
'#,. 0. &t. #, years@ she has *een on digo)in ,' mg 8 hydrochlorothia9ide '! mg daily for chronic
congestive heart fail$re. (ow she has increasing dyspnea. >a* finding:
-eamoglo*in 1,.' gmL1!! ml
CE(<1# mgL1!! ml
(a<12' mEBL>
K<2.1 mEBL>
;l<D! mEBL>
-;O2<2! mEBL>
;-I ray shows evidence of p$lmonary congestion and E;7 shows an atriel rate of ,'!Lmin@ varia*le
/-A *lock and a ventric$lar rate of 1!!Lmin. management. .
-;ontin$e digo)in 8 increase hydroclorothia9ide to '! mgLday
-;ontin$e *oth the :igo)in and hydrochlorothi9ideand *egin 0$rosenamid 5! mg daily.
-3top :igo)in 8 -ydrochlorothia9id and *egin potassi$m s$pplements
-;ontin$e digo)in 8 stop hydrochlorothia9ide 8 add K s$plements
-3top all dr$gs 8 immediatly carry on electrical rhythm reversion

'#2. Which of the following a*normalities is not tr$e parado)ical:
-&arado)ical respiration
-&arado)ical Em*olism
-&arado)ical &$lse
-&arado)ical 3plitting of the ,nd -eart so$nd
'#5. / child with known N$venile polyposis. What will *e the most likely complication .
-:iarrhea
-&ainless *leeding
-&ainf$l *leeding
-;ancer
-(one of the a*ove
'#'. /ll are associated with oligohydramnios EI;E&%:
-4enal agenesis
-&olycystic kidney
-6eningiocele
-&otter's syndrome
-/mniom nodosom
'#?. (eonate 1 month with pne$monia. What is the most likely ca$sative agent .
-E. ;oli
-7C3
-3. &ne$moniae
--. Infl$en9a
-(. 6eningetidis
'#. &t. &resent with conv$lsion after she has *een on IA fl$id for a long time. Investigation shows
(a<1,!
K<2
-;O2<1?
What will *e the *est treatment .
-(ormal saline
--ypertonic saline H di$retics
-'" de)trose
-0$rosemide
-&henytoin
(C: 6;;R ,!!1 F(&11G
'##. Oo$ can do />> 4egarding sho$lder dystocia EI;E&%:
-O)ytocin
-0le) the mothers leg
-Creak the clavicle
-4otate the sho$lders
-3$prap$*ic press$re
'#D. %he most common complication in Epid$ral anasthesia is:
--ypotension

'D!. /ll are side effects of C agonists d$ring preterm la*or EI;E&%:
/c$te &$lmonary Edema
-%achycardia
-/ngina
-Increase of glycemia
-(one of the a*ove
'D1. &t. ;omplains of pain in the >>R. 8 fever. &E: slight a*d tenderness. What do $ do to confirm
the diagnosis .
-Ciopsy
-Cari$m enema
-&eritoneal >avage
-4ectosigmoidoscopy
-E3
(C: diagnosis Is divertic$litis
'D,. Known alcoholic arrived to E4 with large *ilio$s vomiting@ then he vomited large amo$nt of red
*lood 8 complains of pain. What is the most likely diagnosis .
-&erforated $lcer
-Esophageal 1$nction r$pt$re
-4$pt$red Esophageal varice
-/c$te gastritis
(C: F6allory-Weiss tearsG
'D2. /fter yo$ ins$re vital f$nction F/C;G what will *e yo$r initial step in the management. of a
known alcoholic pt. *ro$ght in coma .
-Ait. C1 FthiamineG
->ora9epam
->ook for the s$*d$ral heamatoma
-R$ick control of glycemia Ffinger prickG
-/dminister K
'D5. What is the most imp. 3ide effect of ;>O6I&-E(E .
-Ovarian cyst
-3tim$lation of ov$lation
-%reatment of anov$lation
-/nti Estrogen

'D'. In all there is increased IgE. EI;E&%:
-/sthma
-;hronic *ronchitis
-/llergic rhinitis
-Ec9ema
'D?. In /c$te colicystitis what is the pathophysiology .
-Cacterial invasion of gall*ladder
-O*str$ction of the common *ile d$ct
-O*str$ction of the cystic d$ct
-O*str$ction of the common hepatic d$ct
'D. With -&A infection yo$ may find:
-&$r$lent discharge
-(onp$r$lent discharge
-;ondyloma acc$minata
-;ondyloma lata
'D#. 3t$dy shows that specific mortality rate for stomach ;/ tends to decrease. What is the most
likely e)planation .
-:ecreased &revalence
-:ecrease Incidence
-Cetter screening
-Cetter treatment
'DD. /ll the following will help yo$ to s$spect menopa$se in a ' years lady EI;E&%:
-Aaginal m$co$s atrophy
--%
-0l$sh
-3leep dist$r*ances
-Osteoporosis

?!!. /ll are indications of chorion villo$s sampling EI;E&%:
-%ay-3achs disease
--ygroma ;oli
-%$rner syndrome
-%ranslocation
-6yelomeningeocele
?!1. /ll the following can ca$se *loody diarrhea EI;E&%:
-Oersinia Enterolitica
-E. ;oli
-3higella
-;ampylo*acter
-7iardia lam*lia F4ocky 6.G
?!,. Which germ is fo$nd in $ncooked eggs .
-E. ;oli
-3taph. /$re$s
-3higella
-7iardia
-3almonella
?!2. 2, years old lady with agitation@ hyperthermia@ palpitation @ loss of weight
/nd sweating. What is the initial test to do .
-%hyro)in F%5G
-%3-
-%hyroid scintigraphy
-%hyroid EL3
-%riiodothyronin F%2G
?!5. &ostpart$m &t. Wishes to stop lactation for personal reasons @ how will yo$ prevent the pain
res$lting from *reast engorgement.
-&rescri*e O)ytocin
-/pply *andage to protect clothes
-6edical treatment is $seless most of the time
-/ntiprolactin

?!'. 5' years old male 2 months after car accident@ presents with agitation@ insomnia and
nightmares. -e is an)io$s and there is deterioration in the se)$al relationship with his wife. Whats
yo$r diagnosis.
-Impaired intimacy with his wife
-:epression
-6asked an)iety
-&%3:
-3e)$al dysf$nction
?!?. 0emale ?! years old with constipation for many months@ she has not passed stools for the last 2
days. On e)amination the a*domen is distended *$t non tender. What is yo$r management .
->a)ative
--igh fi*er diet
-6ineral oil
-Enema
?!. 0ollowing a g$n shot a &t. &resents with a swollen right leg@ distal p$lse is weak@ veins are
distended. What is the likely ca$se.
-Enilateral vein throm*osis
-/rterial em*oli
-0emoral nerve in1$ry
-&ost tra$matic s$perficial vein ins$fficiency
-/rterioveno$s fist$la

?!#. 0emale ?! years old on m$ltiple dr$g treatment for *reast cancer with *one metastasis. >ately
she had severe pain and increased dose of opioid ) 2@
now she presents with constipation @lethargy @and apathy. Whats the likely diagnosis.
-7astroenteritis
-7astrointestinal metastasis
-:r$g into)ication
-0ecal impaction

?!D. /lpha 0etoprotein is increased in all of the following e)cept.
-Omphalocele
-&otter syndrome
-/nencephaly
-6eningomyocele
-3pina *ifida
?1!. Which test will help to o*1ectively diagnose fetal heart *eat at 1? weeks .
-E3
-(3% Fnon stress testG
-:oppler
-/*dominal )-ray
?11. 0emale 1# years old complaining of primary amenorrhoea@ Ch;7 is negative .Which test will
yo$ do to diagnose an hypothalmopit$atary disease.
-&elvic e)amination
-&rolactin and %3-
-;% of the sella t$rsica
-&rovera challenge test
-03- and >-
?1,. 4egarding symmetrical Intra$terine growth restriction @ what is the most likely ca$se.
-Intra$terine infection
-6aternal renal disease
-6aternal dia*etes
-:r$g a*$se
?12. 4he$matoid factor will *e fo$nd in all of the following EI;E&%:
-4/ 8 N$venile 4/
-3>E
-31ogren's disease
-:ermatomyositis
-(one of the a*ove
?15. / child with hepatosplenomegaly and mental retardation@ he has positive red$cing s$*stances in
the $rine @:iagnosis.
-7alactosemia
-;ystin$ria
-Aon 7ierke disease
-0r$ctose intolerance
?1'. ;hild 12 years old@ o*ese with painf$l right knee and right hip@ he is limping. On e)amination
there is red$ced a*d$ction and internal rotation@ diagnosis.
-N$venile arthritis
-3eptic arthritis
-/vasc$lar necrosis of the femoral head
-3lipped femoral capital epiphysis
?1?. / known alcoholic man admitted in the E 4 with fever @co$ghing off prof$se and p$r$lent
sp$t$m . On e)amination ronchi on the right lower l$ng@ diagnosis .
->o*ar pne$monia
->$ng a*scess
-;O&:
-%C
?1. / lady with occasional severe attack of dyspnea with facial edema. On e)amination severe
respiratory distress@ red and inflamed pharyn). what is yo$r management .
-/drenalin for o$t pt. $sage
-;ortison I.A
->ateral )- 4ay of laryn)
-Epinephrine 3; Fs$*c$taneo$sG
-Int$*ation

?1#. / carpenter has diffic$lties holding things with hypoesthesic tro$*le of fingers of the the 4t.
-and. Which e)am will help $ to esta*lish the diagnosis .
-Iray of the cervical spine
-Iray of hand
-E67
-E)amination of the rotator c$ff
-(e$rologic e)am of the hand
?1D. 0. ! years present with m$scle weakness with dysphagia 8 change of the skin color of the
hand ind$ced *y cold weather@ with telangiectasia. :iagnosis. .
-:ermatomyositis
-0i*romyositis
-3clerodermia
-3>E
?,!. /siatic man recently came as immigrant to ;anada. -e took chloroB$ine 5 wks 8 , days *efore
his arrival Fto treat his 6alaria infectionG. , years later he develops fever of $nknown origin. What is
the most likely ca$se .
-6alaria reinfection
-1st episode of malaria was not treated s$fficiently
-%he strain was resistant to treatment
-4emnant active schi9ontes in the liver
?,1. /ll the following are characteristics of a narcotic into)ication EI;E&%:
-6yosis
-Cradycardia
-:ry mo$th
-;onstipation
-Orthostatic hypotension
?,,. &t. 2 days postpart$m develops Endometritis. Which is not a finding in this case .
->ow a*dominal pain
-Olig$ria
-%achycardia
-&$r$lent lochia
-Eter$s *elow symphysis
?,2. (ew*orn 2 wks. -ypotonic 8 totally constipated@ his a*dominal seems distended *$t not
painf$l. :iagnosis. .
--irschpr$ng disease
--ypothyroidism
-Aolv$los
-Int$s$sception
-6ilk intolerance
?,5. What is the sign which allows yo$ to make a difference *etween -ankylosing spondylitis 8 4/.
-Noint deviation
-Noint nod$les
-Noint eff$sion
-3acroiliac involvement
?,'. ;hild D years *ro$ght *y his mother@ present headache@ na$sea 8 vomiting. -e feels tired 8
easily falls asleep. :iagnosis. .
-6igraine
-;ere*ral t$mor
-%emporal epilepsy
-3im$lation FfakingG
?,?. /ll are characteristics of /l9heimerMs disease EI;E&%:
-/phasia@ /pra)ia@ /gnosia
->oss of acB$isition of recent facts
->oss of a*stract thinking
->oss of conscio$sness of his environment
->oss of old imp. facts
?,. /ll may ca$se epiglotitis in a child EI;E&%:
--emophil$s infl$en9a
-%o*acco smoke
-&arainfl$en9a vir$s
--ydrocar*$re ingestion
-/c$te allergy
?,#. %he most common ca$se of && Fprecocio$s p$*ertyG in a girl is:
-7onadic t$mor
-Aon 4eclingha$sen disease
-Idiopathic &&
-/drenogenital syndrome
-&it$itary %$mor
?,D. Which is the commonest ca$se of mortality in children aged ' / D years in ;anada .
->e$kemia
-;ancer
-/ccident
-;ongenital malformation
?2!. ;omparing characteristics of Infectio$s monon$cleosis in a child As. /d$lt@ which of the
following is tr$e .
-Its more severe in children
-%ypical form is rare in children
-Imm$ni9ation is *etter
-:is. last longer
-(eed to do throat c$lt$re 8 sensitivity
-6ono spot test is negative.
?21. What will *e the effect of increasing the prevalence of a disease for a screen test .
-Increase in the sensitivity of the test
-Increase in the specificity of the test
-Increase in the &&A.
-Increase in the (&A.
-Increase in the sensitivity 8 &&A. of the test
?2,. Which is contraindicated for forceps delivery .
-%he presenting part at station -1
-2rd degree laceration
-Creech presentation
->arge cap$t
?22. &t. In la*or after delivery of 1st twin has the other twin in verte) presentation 8 at station -1@
mem*. are r$pt$red. %here is no contraction for ,! min$tes. What will $ do .
-O)ytosin drip
-;-section
-0orceps
-Aac$$m
-Wait

?25. 0. ;ame from 7reece complaining from fatig$e 8 weakness@ she is pale. Investigation shows
-*.< 1! gm L dl.
6;A<!
What are yo$ going to do ne)t to reach the diagnosis .
-Cone marrow aspiration
-4etic$locyte level
--*. Electrophoresis
-3er$m iron level
-%rial treatment with C.1,
?2'. &t. 1 years with factitio$s disorder. /ll can *e fo$nd EI;E&%:
-0amily history of similar episode
-%he pt. knows a*o$t medical treatment
--e wishes to attract attention on him
--e has a partic$lar interest in medical profession
-Works in medical profession
?2?. &t. ,2 years with single thyroid nod$le. What will make $ s$spect malignancy .
-%hyroid E3
-;old nod$le on thyroid scan
--ot nod$le on thyroid scan
-;% of the cervical spine
?2. /n infant present with inflamed pharyn) 8 fever. -e was treated with anti*iotics. / week later
he presents again with trism$s. :iagnosis. .
-%etan$s
-&haryngitis
-Infectio$s monon$cleosis
-&eritonsillar a*scess
-3treptococal tonsillitis
?2#. Why is chorionic vilo$s sampling not commonly done in ;anada .
-Its e)pensive
-:octors dont have eno$gh e)perience
-4es$lts are not relia*le
-Its dangero$s
-Ceca$se the prevalence of the diseases that it can detect is low
?2D. /ll can ca$se mania EI;E&%:
-&henel9ine
-%ra9odone
-6ethyphenidate
-;ortisone
->evodopa
?5!. / 5 years old pt. With whitish asymptomatic $rethral discharge. %he investigation of the germ
will reveal :
-;hronic prostatitis and gonococcal $rethritis
-(on specific $rethritis
-Erinary infection
-/c$te prostatitis
?51. What is the allowed amo$nt of 7(& Fgross national prod$ctG to health in ;anada .
-5" / #"
-# " / 1, "
-1, " / 1' "
-1? " / ,5 "
-,' " / 2? "
?5,. -yperglycemic &regnant &t. with level D.# gmL>. after O7%% Foral gl$cose tolerance testG with
'! gm. Which of the following sho$ld not *e done .
-&rescri*e chlorpopramide
-7ive Ins$lin
-/dminister C *locker
-:o a O7%% at ' gm. at , h.
-:iet
(C: screening with '! gm O7%% :
Z If *lood gl$cose *etween .# 8 1!.2 mmolL> then repeat with , hrs. ' gm. O7%%
Z If *lood gl$cose more than 1!.2 mmolL> then diagnosis of dia*etes is made ...
Z Oral hypoglycemics are contraindicated in pregnancy
?52. ;hild ? years old *ro$ght *y his mother with fever and red throat@ lips fiss$res @cervical
adenopathies and rash of e)tremities. Oo$ fo$nd a fine peeling at the hands@ palms and soles of the
feet. yo$ will think of:
-3treptococcal infection
-4$*eolla
-Infectio$s monon$cleosis
-Kawasaki disease
-6easles
?55. ;hild 1, months old adopted from eastern E$rope. -e presents with loss of weight and m$ltiple
adenopathies pl$s splenomegaly associated with skin lesion. What is the diagnosis.
-&roteinoLcaloric maln$trition
-/c$te le$kemia
-Infection with pne$mocystis carinii
-;hild a*$se
--IA infection

?5'. 0orm 1D1 / 1DD, life e)pectancy for men has increase from ?# / 2 years @women 2-#1 years.
What is the most likely ca$se of these changes .
-Aaccination
-Cetter management of ischemic diseases
-Ese of anti*iotics
-Improvement of life style
-7enerali9ation of health care
?5?. /ll the following are a part of diagnosing le$kemia in children EI;E&%:
-%hrom*ocytopenia
-/nemia
--emoglo*in$ria
-7ran$locytopenia
-(e$tropenia
?5. Oo$ can do all of the following investigations in a pregnant pt. &ost term EI;E&%:
-EL3 for e)amination of amniotic fl$id
-/mniocentesis for dosage of alpha fetoprotein
-C&&
-0etal *iometry
-(on stress %est
?5#. 0. '' years old presents for 1 month memory tro$*le pl$s hall$cination@ agitation@ hypotension@
and hot flashes. 3he has vis$al tro$*le with p$pils fi)ed in mydriasis@ what is the most likely
ca$sative s$*stance .
--aloperidol
-Cen9tropin
-%;/
-0l$)otin
-&henel9ine
?5D. Epon the l$ng cancers which one is the most sensitive to radiotherapy.
-:ermoid carcinoma
-3B$amo$s cell carcinoma
-/denocarcinoma
-3mall cell carcinoma
-Endifferentiated small cell carcinoma
?'! which of the following is the main complication of dermoid ovarian cyst .
-Infection
-(ecrosis
-cancerisaton
-%orsion
?'1. Which of the following is a characteristic of a person with somati9ation tro$*le .
-%he person knows that he has psychological tro$*le
-6$ltiple somati9ation tro$*le
--ypochondria
-0eeling of malformation
-Knows that his tro$*les are not tr$e *$t he canMt avoid them
?',. WhatMs the *est criteria for estimating pop$lation health.
-/mo$nt of money spent on health
-7:&
-4atio of physicians per ha*itant
-($m*er of hospitals
?'2. What is the *est way to appreciate health care B$ality d$ring pregnancy and delivery in
;anada.
-(eonatal mortality rate
-&erinatal mortality rate
-Infant mortality rate
-6aternal mortality rate
?'5. 4egarding child schi9ophrenia which is tr$e .
-3igns are identical to the ad$lts
-4isk is 1! " if one of the parents has schi9ophrenia
-&ronostic is good if it appears early
-;omplete presentation from the *eginning
--all$cinations are more freB$ent
?''. / co$ple went for holiday in 6e)ico@ they severe diarrhea. :iagnosis. .
-E. ;oli
-7iardia lam*ia
-Entero*acter
-3almonellosis

(C. %he treatment of choice for travelerMs diarrhea ca$sed *y Enteroto)igenic E. ;oli is :
;yproflo)acin.
?'?. &t. , years. Known to have manic tro$*le for many years@ he is *ro$ght to the E4. /gitated 8
E)cited. What is the most appropriate treatment .
-;ar*ama9epin
-Imipramine
-;lona9epam
->ora9epam
-&henel9ine
?'. /ll are part of K/W/3/KI EI;E&%:
-;oronary heart disease
->ips fiss$res
-&eeling of e)tremities
-7enerali9ed adenopathy
-3traw*erry tong$e
?'#. &t. &resents with l$m*ar pain after lifting a heavy o*1ect. >$m*er I 4ay shows lesions at >'-31
level. Waht will $ find on e)amination .
-/*sent knee 1erk refle) 8 paresis of gastrocnemi$s m$scle
-/*sent knee 1erk refle) 8 achiles refle) 8 paresis of B$adriceps
-/*sent achiles Fankle 1erkG refle) 8 paresis of gastrocnemi$s m$scle
-/*sent achiles Fankle 1erkG refle) 8 paresis of peroneal m$scle
?'D. C agonist $sed for preterm la*or@ may ca$se all EI;E&%:
--eadache
-4espiratory distress
-%achycardia
--yperglycemia
-/ngina
(C: to *e checked

??!. Which of the following ;/ is ca$sed *y vinyl chloride .
->$ng ;/
-Esophag$s ;/
-4hinopharyn) ;/
->iver ;/
-Cladder ;/
??1. &roparonol may *e given in />> the following EI;E&%:
--ypertension
--yperglycemia
->ithi$m ind$ced tremors
-&anic disorder
-;hest /ngina
7ood >$ck in the real e)am ..
:one *y [
#r. thir !hanim
#r. $oger
#r. %. &.
#r. Tariq l'(ubaty

Anda mungkin juga menyukai